СДЕЛАЙТЕ СВОИ УРОКИ ЕЩЁ ЭФФЕКТИВНЕЕ, А ЖИЗНЬ СВОБОДНЕЕ

Благодаря готовым учебным материалам для работы в классе и дистанционно

Скидки до 50 % на комплекты
только до

Готовые ключевые этапы урока всегда будут у вас под рукой

Организационный момент

Проверка знаний

Объяснение материала

Закрепление изученного

Итоги урока

«Элементы теории вероятности для 10-11 классов средней школы»

Категория: Математика

Нажмите, чтобы узнать подробности

Во многих регионах страны с 2004 года были введены в средних школах профильные классы, ориентированные на индивидуализацию обучения и социализацию обучающихся, с учетом реальных потребностей рынка труда. Создание физико-математических и естественно-математических классов средней школы тесно затронуло стороны физического образования и сферу математического образования. Преподавание математики в профильных классах значительно отличается от преподавания в общеобразовательных классах. Одной из важнейших сторон модернизации современного математического образования является включение в школьные программы элементов теории вероятностей. Изучение теории вероятности играет важную ролью в общеобразовательной подготовке современного человека. Обладая вероятностно-статистической грамотностью, современному человеку намного проще и легче будет воспринять социальную, политическую, экономическую информацию. Все современные науки, такие как физика, биология, химия, а также весь блок социально-экономических наук, построены и развиваются на вероятностно статистической базе. И без получения этих знаний невозможно полноценное усвоение вышеперечисленных дисциплин уже в школе.

Цель - теоретически обосновать и содержательно представить цикл уроков и элективный курс по теме «Элементы теории вероятностей» для 10-11 классов средней школы.

Объект исследования - процесс обучения элементам теории вероятностей на урочных и элективных занятиях в средней школе.

При введении теории вероятности в основной курс школы существует проблема в школьных учебниках. И реализация изучения темы теории вероятности в школьных учебниках предоставлена по-разному, а по некоторым вопросам в учебных пособиях не совсем удачно.

Дело в том, что события менее наглядны, чем фигуры, числа или выражения, а шансы и изменчивость не так интуитивны, как длина, площадь или объем. Событие и его шансы – особые типы мыслимых объектов, формализация которых в математические происходит значительно сложнее, чем формализация рисунка или количества. Что скрывать, большинству детей до определенного возраста нет возможности представить об изменчивости и неустойчивости явлений.

Просмотр содержимого документа
««Элементы теории вероятности для 10-11 классов средней школы»»

ЧОУ ДПО «Центр Знаний»



Программа профессиональной переподготовки

«Учитель математики. Теория и методика преподавания учебного предмета «Математика» в условиях реализации ФГОС ООО, ФГОС СОО»






ДИПЛОМНАЯ РАБОТА


на тему: «Элементы теории вероятности для 10-11 классов средней школы»




Выполнил:

Шаронова Селена Михайловна, учитель физики, математики,

ГБОУ Школа №438


г. Санкт-Петербург 2022

Содержание

Введение 3

1 История развития теории вероятности 5

2 Основные понятия теории вероятности 9

3 Методика обучения теории вероятности в 10-11 классах

средней школы 14

4 Разработка элективного курса и тематических уроков 17

Заключение 65

Литература 67

Приложение 69



















Введение

Во многих регионах страны с 2004 года были введены в средних школах профильные классы, ориентированные на индивидуализацию обучения и социализацию обучающихся, с учетом реальных потребностей рынка труда. Создание физико-математических и естественно-математических классов средней школы тесно затронуло стороны физического образования и сферу математического образования. Преподавание математики в профильных классах значительно отличается от преподавания в общеобразовательных классах. Одной из важнейших сторон модернизации современного математического образования является включение в школьные программы элементов теории вероятностей. Изучение теории вероятности играет важную ролью в общеобразовательной подготовке современного человека. Обладая вероятностно-статистической грамотностью, современному человеку намного проще и легче будет воспринять социальную, политическую, экономическую информацию. Все современные науки, такие как физика, биология, химия, а также весь блок социально-экономических наук, построены и развиваются на вероятностно статистической базе. И без получения этих знаний невозможно полноценное усвоение вышеперечисленных дисциплин уже в школе.

Цель - теоретически обосновать и содержательно представить цикл уроков и элективный курс по теме «Элементы теории вероятностей» для 10-11 классов средней школы

Задачи - проанализировать современные тенденции в исследованиях, посвященных вопросам введения в школьную математику элементов теории вероятностей; представить практический материал – решение задач по данной теме, с выработанными методическими указаниями и рекомендациями; разработать структуру, содержание и методику проведения элективного курса «Элементы теории вероятностей» в старших классах средней школы; разработать несколько уроков и провести апробацию.

Объект исследования - процесс обучения элементам теории вероятностей на урочных и элективных занятиях в средней школе.

При введении теории вероятности в основной курс школы существует проблема в школьных учебниках. И реализация изучения темы теории вероятности в школьных учебниках предоставлена по-разному, а по некоторым вопросам в учебных пособиях не совсем удачно.

Дело в том, что события менее наглядны, чем фигуры, числа или выражения, а шансы и изменчивость не так интуитивны, как длина, площадь или объем. Событие и его шансы – особые типы мыслимых объектов, формализация которых в математические происходит значительно сложнее, чем формализация рисунка или количества. Что скрывать, большинству детей до определенного возраста нет возможности представить об изменчивости и неустойчивости явлений.

Школьное содержание в современном мире вплотную приближено к вопросам страхования, торговли, банковских услуг, медицинского обслуживания, управления и принятия решений, к погрешностям измерений любого рода – к явлениям, известным школьникам или их родителям из повседневного опыта, явлениям, значимость которых не вызывает сомнений. Главный упор при изучении теории вероятности нацелен на ясном понимании учащимися концепций изменчивости, средних, рассеивания, выборочных исследований, случайных величин, закона больших чисел. Многие понятия и факты становятся доступны школьникам через эксперимент, а не с помощью дедуктивных выводов и формул. При формировании системы базовых понятий и интуиции нужно, школьники на уроках математики должны размышлять, обсуждать и спорить, можно больше разбирать значимые для школьника сюжеты из жизни. Более остро встает вопрос о необходимости в использовании практико-ориентированного подхода с акцентом на понимании вероятностных ситуаций и описании изменчивости, а не на количественных отношениях между вероятностями. Такой подход используется при решении задач по вероятности в подготовке к ЕГЭ.

Поэтому, считаю, что преподавание и изучение «Теория вероятностей» в средней общеобразовательной школе является насущной необходимостью, обусловленной реалиями современного мира.


1 История развития теории вероятности

У истории теории вероятности не было античных или средневековых предшественников, она целиком — создание Нового времени. Долгое время теорию вероятностей считали чисто опытной наукой и «не совсем математикой». В 1929 году разработали строгое обоснование. В наши дни теория вероятностей занимает одно из первых мест в прикладных науках по широте своей области применения.

Теорию вероятностей можно описательно определить, как математическую теорию случайных явлений.

В повседневной жизни мы часто пользуемся словами «вероятность», «шанс» и т.д. «К вечеру, вероятно, пойдет дождь», «вероятнее всего, мы на всю неделю поедем в деревню», «это совершенно невероятно!», «есть шанс, что успешно сдам экзамен», «всё когда-нибудь действительно произойдет», «чем сильнее неприятность, тем сильнее вероятность» и т.д. - все эти выражения как-то оценивают вероятность того, что произойдет некоторое случайное событие.

Вероятность математическая – числовая характеристика степени возможности появления какого-либо определенного события в тех или иных определенных, могущих повторятся неограниченное число раз условиях. Во второй половине XIX века вероятность вошла в физику в процессе разработки молекулярно-кинетической теории. Вероятностные представления были введены в физику при разработке молекулярно-кинетической теории газов Максвеллом при выводе закона распределения молекул по скоростям и опубликованы в начале 1860 года. Данная публикация отражала содержание его доклада на заседании Британской ассоциации наук 21 сентября 1859 года, посвященного исследованиям в области молекулярно-кинетической теории газов. Позже эти представления более широко использовались Больцманом. Завершением работ Больцмана стала статистическая механика Гиббса, положенная в основу всей статистической термодинамики.

Понятие вероятности разрабатывается наукой уже в течении столетий, а многие ученые-исследователи указывают на его незавершенность и неясность. «Все говорят о вероятности, но никто не может сказать, что это такое».

С вероятностными представлениями мы встречаемся еще в античности. У Демокрита, Лукреция Кара и других античных ученых и мыслителей мы находим глубокие предвидения о строении материи с беспорядочным движением частиц (молекул), встречаем рассуждения о равновозможных исходах (равновероятностных) и т.п. еще в древности делались попытки сбора и анализа некоторых статистических материалов – все это создавало почву для выработки новых научных понятий, в том числе и понятия вероятности. Но античная наука не дошла до выделения этого понятия.

В средневековье мы наблюдаем разрозненные попытки осмыслить встречающиеся вероятностные рассуждения. В работах Л. Пачоли, Н. Тарталья и в первую очередь Д. Кардано уже делались попытки выделить новые понятия – отношения шансов – при решении ряда специфических задач, прежде всего комбинаторных.

Исторически было выделено пять периодов развития теории вероятности. 

  1. До XVI века включительно. В Средневековье натурфилософы ограничивались метафизическими рассуждениями о происхождении случайности и её роли в природе. Математики в этот период рассматривали и иногда решали задачи, связанные с теорией вероятностей, но никаких общих методов и тематических понятий ещё не появилось. Важное достижение - развитие комбинаторных методов. 

  2. Во второй половине XVII века сложились основные понятия, методы теории вероятностей для случайных величин с конечным числом значений. Область применения теории вероятностей в данный период времени начинает расширяться, включая в себя прикладные задачи демографической статистики, страхового дела и теории приближённых вычислений. Важный вклад в идеи новой науки внесли Паскаль и Ферма. Гюйгенс ввёл два фундаментальных понятия: числовая мера вероятности события и понятие математического ожидания случайной величины.  

  3. В XVIII веке стали появляться монографии с систематическим изложением теории вероятностей. Первая книга Якоба Бернулли «Искусство предположений» (1713 год): было предложено классическое определение вероятности случайного события как отношение числа равновероятных исходов, связанных с этим событием, к общему числу исходов. Он изложил правила подсчёта вероятности для сложных событий и дал первый вариант ключевого «закона больших чисел», разъясняющего, почему частота события в серии испытаний не меняется хаотично, а в некотором смысле стремится к своему предельному теоретическому значению (то есть вероятности).  

  4. В начале XIX века: Лаплас, Гаусс, Пуассон:

- применение вероятностных методов в прикладной статистике значительно расширилось,

- понятие вероятности определено для непрерывных случайных величин, - появилась возможность применения методов математического анализа,

- первые попытки применения теории вероятностей в физике,

- появилась статистическая физика,

- строгая теория ошибок измерения,

- вероятностные методы проникают различные прикладные науки.

  1. В XX веке в физике создана теория микромира, а в биологии — теория наследственности (обе основаны на вероятностных методах). Карл Пирсон разработал алгоритмы математической статистики, широко и повсеместно применяемые для анализа прикладных измерений, проверки гипотез и принятия решений. А. Н. Колмогоров дал классическую аксиоматику теории вероятностей. Из новых областей применений теории вероятностей - теория информации и теория случайных процессов.


Теория вероятности возникла как наука в средние века и первая попытка математического анализа состояла в азартных играх (орлянка, кости, рулетка). Эти игры с незапамятных времен создавались рядом поколений именно так, чтобы в них исход опыта был независим от поддающихся наблюдению условий опыта, был чисто случайным. Теория же вероятностей, возникшая в середине XVII века, была ознаменована первыми работами по теории вероятностей, принадлежащие французским учёным Б. Паскалю и П. Ферма и голландскому учёному Х. Гюйгенсу. Теория вероятностей изучает объективные закономерности массовых случайных событий. Она является теоретической базой для математической статистики, занимающейся разработкой методов сбора, описания и обработки результатов наблюдений. Теория вероятностей — раздел математики, изучающий случайные события, случайные величины, их свойства и операции над ними. Для чего нужна теория? Эта теория обеспечивает использующего её концептуальными структурами; в теории происходит разработка терминологии; теория позволяет понимать, объяснять или прогнозировать различные проявления объекта теории, теория предсказывает появление определенных факторов. Знакомство с историей становления и развития теории вероятностей позволит ученикам понять предмет и источники становления математики. Основными стимулами возникновения и первоначального развития теории вероятностей были статистика, страховые общества, философия и азартные игры. Современный этап развития теории вероятностей связан с внутренними потребностями как самой математики, так и других наук.


2 Основные понятия теории вероятности

Теория вероятностей изучает объективные закономерности массовых случайных событий. Она занимается разработкой методов сбора, описания и обработки результатов наблюдений. Путем наблюдений или испытаний, экспериментов, происходит познание явлений действительного мира.

В практической деятельности мы часто встречаемся с явлениями, исход которых невозможно предсказать, результат которых зависит от случая. Случайное явление можно охарактеризовать отношением числа его наступлений к числу испытаний, в каждом из которых при одинаковых условиях всех испытаний оно могло наступить или не наступить.

Теория вероятностей включена в раздел математики, в котором изучаются случайные явления, события и выявляются закономерности при массовом их повторении.

Для полного понимания элементов теории вероятности, необходимо знать некоторые понятия, благодаря которым можно сформировать правильное представление не только о явлениях с ярко выраженной случайностью, но и о таких явлениях, случайная природа которых неочевидна, и затушевана многими осложняющими восприятие факторами.

Вероятность — это степень или относительная мера, количественная оценка возможности наступления некоторого события. Вероятное событие отличается от маловероятного или невероятного, когда какое-нибудь возможное событие произошло в действительности, перевешивает противоположное основание. Также часто вероятность оценивается на качественном уровне, особенно в тех случаях, когда более или менее точная количественная оценка невозможна или крайне затруднена.

Классическое определение вероятности – это понятия равно возможности как объективного свойства изучаемых явлений. Данное свойство является неопределяемым понятием и устанавливается из общих соображений симметрии изучаемых явлений. Наряду с этим понятием имеется понятие элементарного события (исхода), благоприятствующего или нет изучаемому событию. Когда наступление некоторых исключает возможность наступления иных исходов. Это несовместимые элементарные события.

Геометрическое определение вероятности является интуитивно понятным и выведенным из практики, оно не может быть непосредственно применено в случае, если количество равновозможных исходов бесконечно.

Частотное (статистическое) определение вероятности: в некоторых случаях выявить равновозможные случаи может быть невозможно. Формально вероятность можно определить, как предел частоты наблюдений события, предполагая однородность наблюдений (то есть одинаковость всех условий наблюдения) и их независимость друг от друга.

Аксиоматическое определение (аксиоматика Колмогорова) – общепринятая аксиоматика для математического описания теории вероятностей. Аксиоматика Колмогорова позволила придать теории вероятностей стиль, принятый в современной математике (подмножества пространства интерпретируются как случайные события; объединение (сумма) некоторых подмножеств (событий) - событие, заключающееся в наступлении хотя бы одного из этих событий; пересечение (произведение) подмножеств (событий) - событие, заключающееся в наступлении всех этих событий; непересекающиеся множества интерпретируются как несовместные события (их совместное наступление невозможно); в случае пустого множества означает невозможное событие).

Вероятностное пространство — данное понятие было введено в 30-е годы ХХ века А.Н. Колмогоровым для формализации самого понятия вероятности, которое дало начало развитию теории вероятностей, но в виде строгой математической дисциплины. Это тройка множеств: это произвольное множество, элементы которого называются элементарными событиями, исходами или точками; сигма-алгебра подмножеств  , называемых (случайными) событиями; вероятностная мера или вероятность, т.е. сигма-аддитивная конечная мера, такая что 

Классический способ задания вероятностей — это способ задания вероятностей, когда количество элементарных исходов конечно и все они имеют одинаковую вероятность. Тогда вероятность любого события определяется как отношение его мощности (т.е. количества элементарных исходов, благоприятствующих данному событию) к общему числу элементарных исходов:

.

Дискретное вероятностное пространство - если множество элементарных исходов   конечно или счетно:  . В случае дискретных вероятностных пространств событиями обычно считают все возможные подмножества   . В этом случае для задания вероятности необходимо и достаточно приписать каждому элементарному исходу   число   так, чтобы их сумма была равна 1.

Геометрическая вероятность события, являющегося подмножеством множества Ω точек на прямой или плоскости — это отношение площади фигуры A к площади всего множества Ω:

Случайная величина — переменная, значения которой представляют собой численные исходы некоторого случайного феномена или эксперимента. Это численное выражение результата случайного события. Случайная величина является одним из основных понятий теории вероятностей.

Теорема Муавра — Лапласа — одна из предельных теорем теории вероятностей, установлена Лапласом в 1812 году. Данная теорема в теории вероятностей утверждает, что число успехов при многократном повторении одного и того же случайного эксперимента с двумя возможными исходами приблизительно имеет нормальное распределение.

Функция распределения в теории вероятностей — функция, характеризующая распределение случайной величины или случайного вектора; вероятность того, что случайная величина X примет значение, меньшее х, где х — произвольное действительное число. При соблюдении известных условий полностью определяет случайную величину.

Математическое ожидание — понятие в теории вероятностей, означающее среднее (взвешенное по вероятностям возможных значений) значение случайной величины. В случае непрерывной случайной величины - взвешивание по плотности распределения. Математическое ожидание случайного вектора равно вектору, компоненты которого равны математическим ожиданиям компонентов случайного вектора. Некоторые случайные величины не имеют математического ожидания, например, случайные величины, имеющие распределение Коши (класс абсолютно непрерывных распределений, где случайная величина, имеющая распределение Коши, является стандартным примером величины, которая не имеет математического ожидания и дисперсии).

Дисперсия случайной величины — мера разброса значений случайной величины относительно её математического ожидания.

Условная вероятность — наступления одного события при условии, что другое событие уже произошло.

Закон больших чисел (ЗБЧ) в теории вероятностей — принцип, описывающий результат выполнения одного и того же эксперимента много раз. Согласно закону, среднее значение конечной выборки из фиксированного распределения близко к математическому ожиданию этого распределения. Закон больших чисел важен, поскольку он гарантирует устойчивость для средних значений некоторых случайных событий при достаточно длинной серии экспериментов. Данный закон применим только тогда, когда рассматривается большое количество испытаний.

Центральные предельные теоремы (ЦПТ) — класс теорем в теории вероятностей, утверждающих, что сумма достаточно большого количества слабо зависимых случайных величин, имеющих примерно одинаковые масштабы, имеет распределение, близкое к нормальному.

Байесовская вероятность — интерпретация понятия вероятности, которая используется в байесовской теории. Вероятность определяется как степень уверенности в истинности суждения.

Плотность вероятности — это способ задания распределения случайной величины. Во многих практических приложениях понятия «плотность вероятности» и «плотность случайной величины (или функция распределения вероятностей) синонимизируются как вещественная вероятность, которая характеризуется в сравнительную вероятность реализации тех или иных значений случайной переменной (переменных).

Парадокс Монти Холла — одна из известных задач, которая противоречит здравому смыслу. Она не является парадоксом, т.к. её решение может показаться неожиданным. Да и многим людям бывает сложно принять правильное решение даже после того, как его им рассказали.

Линейная частичная информация — теория, применяемая для принятия решений на основании логики нечётной при неполной или неаккуратной доступной информации. Данная теория была изобретена в 1970 году швейцарским математиком Эдуардом Кофлером.


3 Методика обучения теории вероятности в 10-11 классах средней школы

Преподавание предметов, составляющих традиционный курс математики в средней общеобразовательной школе, базируется на ряде элементарных навыков, формирующих фундамент математического образования школьников: решение уравнений, вычисления, решение задач на движение и ряда других. При решении такого рода задач от учащихся, требуется правильное определение и применение правил.

Методика обучения элементам теории вероятностей требует от учителя кардинального изменения стиля своей работы. Она подразумевает организацию дискуссий на уроке, интенсивную устную работу, а также расширение собственного кругозора в областях других наук, таких, как биология, география, история, литература и многих других, в дополнение к сложившимся методам и подходам к обучению. Изучение элементов теории вероятностей могут быть заимствованы из курса геометрии, где необходимо решить несколько задач, абсолютно непохожих друг на друга. Задачи могут быть совершенно не аналогичны, и решение одной из задач не означает, что будет с легкостью решена следующая. Поэтому главное условие в области преподавания теории вероятностей является изменение технологии учительской деятельности при преподавании этого курса.

Аналогичные изменения должны произойти и в позиции ученика. В первую очередь должно измениться его поведение на уроке и при подготовке к нему. Известно, что дети привыкли к определенному стилю преподавания математики, требующему от них умения решать пусть и обширный, но заранее очерченный круг заданий. Они довольствуются тем, что умеют многократно воспроизводить изученный алгоритм и даже противятся попыткам решить задачу другим способом. Большие трудности возникают у учеников при решении нестандартных задач, тем более задач повышенной сложности, в основе которых лежит редко используемый алгоритм. Часто возникают ситуации, когда ученик не готов к собственной интеллектуальной активности и к аргументированному отстаиванию своей точки зрения. Поэтому на уроках и элективных курсах, где изучаются элементы теории вероятностей, требуется создание специальной среды, способствующей этим изменениям, и погружение в нее учащихся. Это конкретно касается проведения практических работ, экспериментов, исследовательской и проектной деятельности непосредственно в ходе урока. Важно активное участие учащихся в дискуссии, а также поиск ими информации за пределами школьного учебника, анализ различных массивов данных с целью выявления закономерностей, самостоятельный выбор инструментария для своей работы. При этом представляется абсолютно необходимым использование в работе на уроке и дома информационно-вычислительной техники, включая ресурсы Internet. Безусловно, эти требования усложняют жизнь и ученику, и учителю.

Подходы, методы, реализуемые при изучении теории вероятности, должны изменить и отношение учеников к случайному событию, которое часто идет вразрез с имеющимися у детей представлениями. Необходимо иметь в виду, что собственный жизненный опыт и фантазия учеников порой только мешают им, уводя в сторону от решения задачи. Богатое воображение учащихся подлежит жесткому ограничению с самых первых уроков, когда вводятся понятия «случайный эксперимент», «исход эксперимента» и ряд других. Из обсуждения должны быть исключены любые фантастические условия проведения случайного эксперимента, и до учащихся должна быть донесена мысль, что «случайно» обозначает вовсе не «все что угодно». На протяжении многих уроков надо формировать понимание того, что ищется закон, который управляет случайными процессами без влияния везения и фантастики, причем стихийно это понимание у учащихся не образуется.

К сожалению, усвоение вероятностных и статистических понятий происходит только на уроках теории вероятностей и нет никакого подкрепления при изучении других школьных предметов. Вероятностное мышление со всем многообразием ожидаемых исходов не присутствует в их содержании. В курсе их изучения по-прежнему царят неизбежность наступления ожидаемого результата и полная предсказуемость всех процессов. Знакомство с современными задачами экономики, рассматривающими обоснование целесообразности освоения новых территорий, строительства промышленных объектов, железнодорожных магистралей, выбора мест для строительства школ, больниц - все это остается за рамками школьного образования. Выпускник школы может и не догадываться, что за всем этим стоит современная математика.

Самым сложным является анализ текста условия задачи. Это особенно проявляется в курсе преподавания теории вероятностей, где все задачи являются сюжетными. В отличие от курса алгебры, где в контрольных работах всегда присутствуют задачи на вычисления, решение уравнений и неравенств, контрольная работа по теории вероятностей содержит исключительно текстовые задачи. Даже несложные вероятностные задачи сводятся к одному или двум комбинаторным приемам, решение которых учащиеся должны освоить максимум за три урока. Сюжетные задачи по теории вероятности гораздо разнообразнее, чем алгебраические. Помимо «классических» задач на бросание кубиков, монет, вытягивание наугад разноцветных карточек, существует огромное число прочих сюжетов. И для ученика часто очень трудно решая «новую» задачу, понять, что это «старая», только что решенная задача, но в «новой упаковке». Увидеть аналогию в задачах на вытаскивание из мешка разноцветных шариков или черных и белых пешек способны только достаточно подготовленные ученики.

4 Разработка элективного курса и тематических уроков

Общая характеристика элективного курса

Элективный курс «Элементы теории вероятности» разработан в рамках реализации концепции профильного обучения на старшей ступени общего образования и соответствует Государственному стандарту среднего образования по математике. При разработке данной программы учитывалось то, что элективный курс как компонент образования должен быть направлен на удовлетворение познавательных потребностей и интересов старшеклассников, на формирование у них новых видов познавательной и практической деятельности, которые не характерны для традиционных учебных курсов.

Теория вероятностей является одним из классических разделов математики. В настоящее время она завоевала очень серьезное место в науке и прикладной деятельности. Ее идеи, метода и результаты не только используются, но буквально пронизывают все естественные и технические науки, экономику, планирование, организацию производства, связи, а также такие далекие, казалось бы, от математики науки, как лингвистику и археологию. Сейчас без достаточно развитых представлений о случайных событиях и их вероятностях, без хорошего представления о том, что явления и процессы, с которыми мы имеем дело, подчиняются более сложным закономерностям, невозможно полноценно работать в физике, химии, биологии, управлять производственными процессами.

Следовательно, данная тема актуальна и нуждается в рассмотрении. Теория вероятностей уделено недостаточно внимания в школьном курсе математики, так как в учебниках очень мало заданий, хотя на ЕГЭ она выделена в отдельную тему. Решением данной проблемы служит создание элективного курса по теории вероятностей.

Программа курса конкретизирует содержание предметных тем образовательного стандарта и дает примерное распределение учебных часов по разделам курса. Курс позволяет школьникам систематизировать, расширить и укрепить знания, подготовиться для дальнейшего изучения тем, научиться решать разнообразные задачи различной сложности.

Цели курса:

- обобщение и систематизация, расширение и углубление знаний по изучаемым темам;

- приобретение практических навыков выполнения заданий, повышение математической подготовки школьников;

- развитие вероятностного мышления учащихся; воспитание понимания значимости математики для научно-технического прогресса;

- сформировать умения решать комбинаторные задачи, посредством перебор возможных вариантов, а также умение использовать изученные формулы;

- закрепить полученные теоретические знания во время практического решения прикладных задач; освоить конкретные математические знания, которые являются необходимыми в применении для продолжения образования.

Задачи курса:

- развивать представления о вероятностно-статистических закономерностях в окружающем мире; развивать логическое мышление;

- совершенствовать интеллектуальную, речевую и письменную культуру путем обогащения математического аппарата.

- подготовить учащихся к итоговой аттестации в форме ЕГЭ.

Методы и формы обучения

Основной тип занятий - практикум.

Для наиболее успешного усвоения материала планируются различные формы работы с учащимися: лекционно-семинарские занятия, групповые, индивидуальные формы работы.

Для текущего контроля на каждом занятии учащимся рекомендуется серия заданий, часть которых выполняется в классе, а часть - дома самостоятельно.

Структура курса представляет собой 6 логически законченных и содержательно взаимосвязанных темы, изучение которых обеспечит системность и практическую направленность знаний и умений учеников, поможет в решении стереометрических задач школьного курса геометрии 10-11 классов. Разнообразный дидактический материал дает возможность отбирать дополнительные задания для учащихся разной степени подготовки. Все занятия направлены на расширение и углубление базового курса. Содержание курса можно варьировать с учетом склонностей, интересов и уровня подготовленности учеников.

Описание места элективного курса в учебном плане

Данная программа рассчитана на два года обучения для учащихся 10-11 классов. Количество часов за учебные года – 68.

Требования к уровню подготовки учащихся

В результате изучения курса «Элементы теории вероятностей» учащийся должен иметь представление:

- о вероятностном характере различных процессов окружающего мира;

- о роли и месте теории вероятностей, комбинаторики и математической статистики при освоении смежных дисциплин по выбранному профилю и в сфере профессиональной деятельности;

- о значении и области применения теории вероятностей, комбинаторики и математической статистики.

Знать/понимать:

- основы комбинаторики и теории вероятностей;

- виды случайных событий и операции над событиями;

- основные формулы для определения вероятности сложных событий;

- основные понятия математической статистики и теории вероятностей;

- различать дискретные и непрерывные случайные величины;

- видеть в конкретных житейских, научных, и технических проблемах вопросы, задачи, которые допускают решение методами теории вероятностей, умение сформулировать и решить такие задачи.

Уметь:

- рассчитывать вероятности событий с применением формулы классической вероятности;

- вычислять вероятность суммы и произведения событий;

- применять геометрическую вероятность при решении задач;

- вычислять вероятности событий, используя различные определения вероятности и формулы;

- представить событие, как комбинация, состоящая из нескольких элементарных событий;

- использовать приближенные формулы для вычисления вероятностей;

- находить числовые характеристики случайных величин;

- решать простейшие задачи математической статистики и интерпретировать полученные результаты;

- анализировать реальные числовые данные, представленные в виде таблиц, графиков, диаграмм;

- строить законы распределения случайных величин;

- вычислять математическое ожидание и дисперсию, и их точечные и интервальные оценки;

- проверять гипотезы о среднем и дисперсии;

- решать комбинаторные задачи.

Формы контроля: тестовые задания

Содержание элективного курса для 10 класса

Раздел 1. «Статистика» (8 часов)

Проведение трех лекций по темам «Среднее арифметическое. Мода. Размах», «Медиана. Дисперсия» и «Статистические исследования», а также разбор задач с подробным решением их у доски. Самостоятельное

решение задач по данной теме.

Итоговая работа по разделу 1: «Свободные колебания».

Раздел 2. «Комбинаторика» (6 часов)

Проведение двух лекций по темам «Правило умножения. Перестановки. Факториал» и «Размещения. Сочетания», а также разбор задач с подробным решением их у доски. Самостоятельное решение задач по данной теме.

Итоговая работа по разделу 2: «Комбинаторика».

Раздел 3. «Теория вероятностей» (10 часов)

Проведение четырех лекций по темам «Случайные события», «Классическое определение вероятности», «Аксиоматический подход к понятию “вероятность”» и «Статистический способ подсчета вероятности», а также разбор задач с подробным решением их у доски. Самостоятельное решение задач по данной теме.

Итоговая работа по разделу 3: «Теория вероятностей».

Раздел 4. «Подготовка к ЕГЭ» (9 часов)

Решение задач по теории вероятностей и комбинаторики из ЕГЭ

Итоговый тест по всему курсу: «Элементы теории вероятностей» (1час).

Содержание элективного курса для 11 класса (34 часа)

Раздел 1. «Опыты с равновозможными элементарными событиями» (10 часов)

Первоначальные понятия теории вероятностей. Понятие случайного события. Совместные и несовместные события. Полная система событий. Равновозможные события.

Раздел 2. «Простейшие правила и формы вычисления вероятностей» (10 часов)

Сумма и произведение событий. Вероятность суммы несовместных событий (теорема сложения вероятностей). Вероятность суммы и произведения совместных событий.

Раздел 3. «Геометрическая вероятность» (6 часов)

Решение задач геометрической вероятности.

Раздел 4. «Решение задач ЕГЭ по математике» (8 часов)

Решение задач № 4 профильного экзамена по математике (Начала теории вероятностей).

Тематическое планирование курса 10 класс (34 часа)

№ п/п

Тема занятия

Количе ство

часов

Раздел 1. «Статистика»

1

Лекция на тему «Среднее арифметическое. Мода.

Размах»

1

2

Разбор задач с решением их у доски

1

3

Лекция на тему «Медиана. Дисперсия»

1

4

Разбор задач с решением их у доски

1

5

Лекция на тему «Статистические исследования»

1

6

Разбор задач с решением их у доски

1

7

Самостоятельное решение задач по всему 1 разделу

1

8

Выполнение итоговой работы по разделу 1: «Статистика»

1

Раздел 2. «Комбинаторика»

9

Лекция на тему «Правило умножения. Перестановки.

Факториал»

1

10

Разбор задач с решением их у доски

1

11

Лекция на тему «Размещения. Сочетания»

1

12

Разбор задач с решением их у доски

1

13

Самостоятельное решение задач по всему 2 разделу

1

14

Выполнение итоговой работы по разделу 2:

«Комбинаторика»

1

9

Лекция на тему «Правило умножения. Перестановки.

Факториал»

1

Раздел 3. «Теория вероятностей»

15

Лекция на тему «Случайные события»

1

16

Разбор задач с решением их у доски

1

17

Лекция на тему «Классическое определение

вероятности»

1

18

Разбор задач с решением их у доски

1

19

Лекция на тему «Аксиоматический подход к понятию

“вероятность”»

1

20

Разбор задач с решением их у доски

1

21

Лекция на тему «Статистический способ подсчета

вероятности»

1

22

Разбор задач с решением их у доски

1

23

Самостоятельное решение задач по всему 3 разделу

1

24

Выполнение итоговой работы по разделу 3: «Теория

вероятностей»

1

Раздел 4. «Задачи по теории вероятностей и комбинаторики из ЕГЭ»

25-33

Решение задач по теории вероятностей и комбинаторики из ЕГЭ

9

34

Выполнение итогового теста по всему курсу «Элементы теории вероятностей»

1


Тематическое планирование курса 11 класс (34 часа)


№ п/п

Тема занятия

Количе ство

часов

Раздел 1. «Опыты с равновозможными элементарными событиями»

1

Первоначальные понятия теории вероятностей

1

2-4

Понятие случайного события

3

5-7

Совместные и несовместные события

3

8-10

Полная система событий. Равновозможные события

3

Раздел 2. «Простейшие правила и формы вычисления вероятностей»

11-13

Сумма и произведение событий.

3

14-16

Вероятность суммы несовместных событий (теорема сложения вероятностей).

3

17-20

Вероятность суммы и произведения совместных событий.

4

Раздел 3. «Геометрическая вероятность».

21-26

Решение задач геометрической вероятности.

6

Раздел 4. «Решение задач ЕГЭ по математике».

27-34

Решение задач № 4 профильного экзамена по математике (Начала теории вероятностей ).

8



Урок-лекция по теме «История развития теории вероятности»


Цель: повторить основные элементы комбинаторики; рассмотреть этапы развития теории вероятностей как науки.

Задачи урока:
1) образовательные:
- обобщить и систематизировать основные понятия изучаемой темы;
- отработать и закрепить практические навыки решения ключевых задач;
2) развивающие:
- продолжить формирование аналитического и логического мышления учащихся;
- продолжить формирование у учащихся навыков самостоятельной деятельности;
3) воспитательные:
- воспитывать коммуникативные компетенции;
- продолжить формирование общей и математической культуры учащихся;
- воспитывать понимание значимости ведущей роли математики в развитии современного научно-технического общества;
- повысить положительную мотивацию к учению.

Результаты обучения: учащиеся закрепляют основные навыки решения задач по данной теме, приобретают навык самостоятельной подготовки.

Форма: обзорная лекция.

Ход урока.

  1. Организационный момент.

  2. Повторение.

    1. Устная работа.

    2. Практическая работа

  3. Новый материал

  4. Домашнее задание

  1. Организационный момент.

Приветствие учеников, сообщение темы и цели урока.

2. Повторение.

2.1. В начале урока мы повторим с вами основные элементы комбинаторики.

Размещение

Это любое упорядоченное подмножество m из элементов множества n.

Перестановки ( ). Если m = n, то эти размещения называются перестановками.

Сочетания ( ) – это любое подмножество из m – элементов, которые принадлежат множеству, состоящему из n – различных элементов.

Следствие. Число сочетаний из n элементов по n – m равно число сочетаний из n элементов по m, т.е. .

2.2. Практическая работа. Разберем несколько задач.

Задача.1. Сколько можно записать четырехзначных чисел, используя без повторения все 10 цифр?

Решение:

1) .

2) т.к. есть среди чисел 0, который не может стоять впереди, поэтому надо еще найти .

3) .


Задача.2. Пусть имеется множество, содержащие 4 буквы: {А,В,С,Д}. Записать все возможные сочетания из указанных букв по три. Решение: Таких сочетаний будет 4: АВС; АСД; АВД; BCД. Здесь в число сочетаний не включены, например, АВС, ВСА, т.к. у нас уже есть АВС, потому что порядок элементов в сочетании не учитываются.


Задача.3. Сколькими способами можно расставить 9 различных книг на полке, чтобы определенные 4 книги стояли рядом?

Решение: если обозначить 4 определенные книги как одно целое, то получается 6 книг, которые можно переставлять.

переставляются, 4 определенные книги можно переставлять . Тогда всего перестановок по правилу умножения будет


Задача.4. Нужно выбрать в подарок 4 из 10 имеющихся книг. Сколькими способами это можно сделать?

Решение:


Задача.5. Имеется 10 белых и 5 черных шаров. Сколькими способами можно выбрать 7 шаров, чтобы среди них были 3 черных.

Решение: .

Белые шары

Черных шаров

Тогда

В размещении учитывается порядок элементов при выборе, а в сочетаниях – не учитывается.


Задача.6. Сколькими способами можно группу из 12 человек разбить на 2 подгруппы, в одной из которых должно быть не более 5, а во второй – не более 9 человек.

Решение:

Первая подгруппа может состоять либо из 3, либо из 4, либо из 5 человек.

, , , .


Задача.7. Десять команд участвуют в разыгрывание первенства по футболу, лучшие из которых занимают 1-е, 2-е и 3-е места. Две команды, занявшие последние места не будут участвовать в следующем таком же первенстве. Сколько разных вариантов результата первенства может будут учитывать, если только положение первых трех и последних 2-х команд?

Решение: 1-е три места может будут распределены: способов.

Остается 7 команд, две из которых выбывают из следующего первенства т.к. порядок выбывших команд не учитывается = способом.

Тогда число возможных результатов = .


Задача.8. Сколько существует вариантов опроса 11 учащихся на одном занятии, если ни один из них не будет вызван дважды и на занятии может будет опрошено любое количество учащихся, порядок опроса не важен.

Решение:

может не спросить ни одного, т.е. ,

если только 1, то ,

если только 2-х то и т.д.

Тогда он всего опросит

3. Новый материал. Проект «Предмет теории вероятностей».

История развития теории вероятностей, а с нею и развитие понятия вероятности можно разбить на следующие этапы.


Д. Кардано 

1. Предыстория теории вероятностей. В этот период, начало которого теряется в веках, ставились и решались элементарные задачи, которые позже будут отнесены к теории вероятностей. Никаких специальных методов в этот период не возникает. Этот период кончается работами Кардано,  Пачоли, Тарталья и др. С вероятностными представлениями мы встречаемся еще в античности. У Демокрита, Лукреция Кара и других античных ученых и мыслителей мы находим глубокие предвидения о строении материи с беспорядочным движением мелких частиц (молекул), мы встречаем рассуждения о равновозможных исходах (равновероятных) и т. п.

Еще в древности делались попытки сбора и анализа некоторых статистических материалов — все это (а также и другие проявления внимания к случайным явлениям} создавало почву для выработки новых научных понятий, в том числе и понятия вероятности.

Но античная наука не дошла до выделения этого понятия. В философии вопрос о случайном, необходимом и возможном всегда был одним из основных. Философская разработка этих проблем также оказывала влияние на формирование понятия вероятности.

В целом в средневековье мы наблюдаем только разрозненные попытки осмыслить встречающиеся вероятностные рассуждения.


Н. Тарталья

2. Возникновение теории вероятностей как науки. К середине, XVII в. вероятностные вопросы и проблемы, возникающие в статистической практике, в практике страховых обществ, при обработке результатов наблюдений и в других областях, привлекли внимание ученых, так как они стали актуальными вопросами. В первую очередь это относится к        Б.Паскалю, П. Ферма и X. Гюйгенсу. СЛАЙД 5. В этот период вырабатываются первые специфические понятия, такие, как математическое ожидание и вероятность (в форме отношения шансов), устанавливаются и используются первые свойства вероятности: теоремы сложения и умножения вероятностей. В это время теория вероятностей находит свои первые применения в демографии, страховом деле, в оценке ошибок наблюдения, широко используя при этом понятие вероятности.

3. Следующий период начинается с появления работы Я. Бернулли "Искусство предположений" (1713), в которой впервые была строго доказана первая предельная теорема — простейший случай закона больших чисел. СЛАЙД14. К этому периоду, который продолжался до середины XIX в., относятся работы Муавра, Лапласа, Гаусса и др. В центре внимания в это время стоят предельные теоремы. Теория вероятностей начинает широко применяться в различных областях естествознания. И хотя в этот период начинают применяться различные понятия вероятности (геометрическая вероятность, статистическая вероятность), господствующее положение занимает, в особенности после работ Лапласа, так называемое классическое определение вероятности.

4. Следующий период развития теории вероятностей связан прежде всего с Петербургской математической школой. За два столетия развития теории вероятностей главными ее достижениями были предельные теоремы. Но не были выяснены границы их применимости и возможности дальнейшего обобщения. Наряду с огромными успехами, достигнутыми теорией вероятностей в предыдущий период, были выявлены и существенные недостатки в ее обосновании, это в большой мере относится к недостаточно четким представлениям о вероятности.

В теории вероятностей создалось положение, когда дальнейшее ее развитие требовало уточнения основных положений, усиления самих методов исследования. Это было осуществлено русской математической школой во главе с П. Л. Чебышевым. Среди ее крупнейших представителей мы видим А. А. Маркова и А. М. Ляпунова. В этот период в теорию вероятностей входят оценки приближений предельных теорем, а также происходит расширение класса случайных величин, подчиняющихся предельным теоремам. В это время в теории вероятностей начинают рассматривать некоторые зависимые случайные величины (цепи Маркова).

Понятие вероятности получило большое распространение в естественных науках, в первую очередь это относится к физике. Появляются работы Максвелла, а затем Больцмана и Д. Гиббса. Их трудами создается статистическая физика. Но это внедрение вероятностных методов и понятий в физику шло в довольно большом отрыве от достижений теории вероятностей.

Развитие теории вероятностей в начале ХХ в. привело к необходимости пересмотра и уточнения ее логических основ, в первую очередь понятия вероятности. Следует иметь в виду и то, что к началу ХХ в. аксиоматический метод стал проникать во многие области математики (работы Д. Гильберта, Пеано и др.), что также оказало влияние на теорию вероятностей. В результате всего этого возникла необходимость аксиоматизации теории вероятностей и ее основного понятия — вероятности.

5. Современный период развития теории вероятностей начался с установления аксиоматики. Этого прежде всего требовала практика, так как для успешного применения теории вероятностей в физике, биологии и других областях науки, а также в технике и военном деле необходимо было уточнить и привести в стройную систему ее основные понятия. Благодаря аксиоматике теория вероятностей стала абстрактно-дедуктивной математической дисциплиной, тесно связанной с другими математическими дисциплинами. Это обусловило небывалую широту исследований по теории вероятностей и ее применениям, начиная от хозяйственно-прикладных вопросов и кончая самыми тонкими теоретическими вопросами теории информации и теории случайных процессов.

Первые работы этого периода связаны с именами С. Н, Бернштейна, Р. Мизеса, Э. Бореля. Окончательное установление аксиоматики произошло в 30-е годы ХХ в. Анализ тенденций развития теории вероятностей позволил А. Н. Колмогорову создать общепринятую аксиоматику.

В этот период понятие вероятности проникает почти во все сферы человеческой деятельности, становясь одним из основных понятий современной науки. Возникают самые различные определения вероятности, несводимые друг к другу. Многообразие определений основных понятий — существенная черта современной науки, и понятие вероятности не исключение.


4. Домашнее задание.

Выполнить решение четырех задач на заданную тему.

1. Сколькими способами 9 человек могут встать в очередь в театральную кассу? (Решение: )

2. На плоскости отметили 5 точек. Их надо обозначить латинскими буквами. Сколькими способами это можно сделать (в латинском алфавите 26 букв)?

(Решение: )

3.В магазине продается 8 различных наборов марок. Сколькими способами можно выбрать из них 3 набора? (Решение: способов.)

4. Сколькими способами из класса, где учатся 24 учащихся, можно выбрать: а) двух дежурных, б) старосту и его заместителя?

(Решение: а) выбор кто на первом месте, кто на втором не важен, поэтому способов в два раза меньше, чем в б) 552:2=276; б) старосту можно выбрать 24 способами, а помощника из оставшихся учеников 23 способами, всего 23*24=552)

Урок по теме «Теория вероятности. Решение задач»

Тема урока: «Теория вероятности. Решение задач.»

Тип урока: комбинированный.

Длительность: 2 учебных часа.

Цель урока: рассмотреть простейшие понятия теории вероятностей.

Задачи урока:

образовательные:

- научить в процессе реальной ситуации определять достоверные, невозможные, равновероятностные, совместные и несовместные события;

- научить решать задачи из жизни;

воспитательные:

- воспитание умения слушать и вступать в диалог, участвовать в коллективном обсуждении проблем, интегрироваться в группу сверстников и строить продуктивное взаимодействие, настойчивости в достижении цели и заинтересованности в конечном результате труда;

развивающие:

- развитие умения анализировать, обобщать изучаемые факты, выделять и сравнивать существенные признаки, выбирать наиболее эффективные способы решения задач в зависимости от конкретных условий;

- рефлексия способов и условий действия; контроль и оценка процесса и результатов деятельности.

Используемые технологии:

- развивающее обучение,

- групповая технология,

- ИКТ,

- элементы исследовательской деятельности, элементы блочного изучения тем.

Оборудование и материалы для урока: компьютер, проектор, презентация по теме «Простейшие вероятностные задачи», экран.

План урока:

1. Организационный момент.

2. Изучение нового материала.

2.1. Что такое событие?

2.2. Что такое теория вероятностей? Алгоритм нахождения вероятности случайного события.

2.3. Решение простейших вероятностных задач.

3. Итоги урока.

4. Домашнее задание.


Ход урока

1. Организационный момент

Приветствие учеников, сообщение темы и цели урока

2. Изучение нового материала

2.1. Что такое событие? (класс заранее был поделен на группы, одна из групп подготовила информацию об этом понятии)

Например, в теории вероятностей возможный исход эксперимента, называется элементарным событием, а множество таких исходов называется просто событием.

Событие – это результат испытания.

Пример.

Стрелок стреляет по мишени, разделенной на четыре области. Выстрел – это испытание. Попадание в определенную область мишени – событие. В урне имеются цветные шары. Из урны наудачу берут один шар. Извлечение шара из урны есть испытание. Появление шара определенного цвета – событие.

В теории вероятностей под событием понимают то, относительно чего после некоторого момента времени можно сказать одно и только одно из двух. Да, оно произошло. Нет, оно не произошло.

В жизни мы постоянно сталкиваемся с тем, что некоторое событие может произойти, а может и не произойти.

Например, в следующем году первый снег выпадет в субботу. Бутерброд упадет маслом вниз. При бросании кубика выпадет шестерка. При бросании кубика выпадет четное число.

У меня есть лотерейный билет. После опубликования результатов розыгрыша лотереи интересующее меня событие – выигрыш тысячи рублей, либо происходит, либо не происходит. В следующем году первый снег выпадет в субботу.

Такие непредсказуемые события называются случайными.

Теория вероятностей изучает различные модели случайных событий, их свойства и характеристики. Разумеется, эта теория не может однозначно предсказать, какое событие в реальности произойдет, но может оценить, какое событие наиболее вероятно. При этом будем считать, что случайные события равновероятны (или равновозможны), - идеализированная модель.


Два события, которые в данных условиях могут происходить одновременно, называются совместными, а те, которые не могут происходить одновременно, - несовместными.

Примеры.

1. Из ящика с деталями наудачу извлечена деталь. Появление стандартной детали исключает появление нестандартной детали. События «появилась стандартная деталь» и «появилась нестандартная деталь» - несовместные.

2. Брошена монета. Появление «герба» исключает появление надписи. События «появился герб» и «появилась надпись» - несовместные.

3. Примеры ребят.


Равновозможными называются события, когда в их наступлении нет преимуществ.

Неравновозможные события те, у которых в наступлении одного из событий есть какое-то преимущество.

Примеры.

1. Появление герба или надписи при бросании монеты представляют собой равновероятные события.

2. Пусть бросают игральную кость. В силу симметрии кубика можно считать, что появление любой из цифр 1, 2, 3, 4, 5 или 6 одинаково возможно (равновероятно).

3. Примеры учеников.


Событие, которое происходит всегда, называют достоверным событием. Вероятность достоверного события равна 1. Событие, которое не может произойти, называется невозможным. Вероятность невозможного события равна 0.


Примеры.

1. В следующем году снег не выпадет. При бросании кубика выпадет семерка. Это невозможные события.

2. В следующем году снег выпадет. При бросании кубика выпадет число, меньше семи. Ежедневный восход солнца. Это достоверные события.

3. Пусть, например, из урны, содержащей только черные шары, вынимают шар. Тогда появление черного шара – достоверное событие; появление белого шара – невозможное событие.

4. Приведите примеры достоверных и невозможных событий.

2.2. Что такое «теория вероятностей»? (ребята из третьей группы знакомят с определениями теории вероятностей)

Например, теория вероятностей – раздел математики, изучающий закономерности случайных явлений: случайные события, случайные величины, их свойства и операции над ними. (Советский энциклопедический словарь, 1982 год)

Теория вероятностей – это математическая наука, позволяющая по вероятностям одних случайных событий находить вероятности других случайных событий, связанных каким – либо образом с первыми. (А.А.Боровков «Теория вероятностей», М.: Наука, 1986 год.)

Вероятность – это численная характеристика реальности появления того или иного события.

Классическое определение вероятности. Вероятностью события А при проведении некоторого испытания называют отношение числа тех исходов, в результате которых наступает событие А, к общему числу всех (равновозможных между собой) исходов этого испытания.


Для решения задач используют алгоритм нахождения вероятности случайного события.

Для нахождения вероятности случайного события А при проведении некоторого испытания следует найти:

число N всех возможных исходов данного испытания;

количество N(A) тех исходов, в которых наступает событие А;

частное оно и будет равно вероятности события А.

Принято вероятность события А обозначать так: Р(А). Значит Р(А) =


Примеры. На завод привезли партию из 1000 подшипников. Случайно в эту партию попало 30 подшипников, не удовлетворяющих стандарту. Определить вероятность Р(А) того, что взятый наудачу подшипник окажется стандартным.

Решение. Число стандартных подшипников равно 1000 – 30 = 970. Будем считать, что каждый подшипник имеет одинаковую вероятность быть выбранным. Тогда полная группа событий состоит из N = 1000 равновероятных исходов, из которых событию А благоприятствуют N(A) = 970 исходов.

Поэтому Р(А) =

Ответ: 0,97.

2. Найдем вероятность того, что при одном бросании игральной кости (кубика) выпадает: а) три очка; б) число очков, кратное трем; в) число очков больше трех; г) число очков, не кратное трем.

Решение. Всего имеется N=6 возможных исходов: выпадение 1,2,3,4,5,6 очков. Считаем, что эти исходы равновозможны.

а) Только при одном из исходов N(А)=1 происходит интересующее нас

событие А – выпадение трех очков. Вероятность этого события .

б) При двух исходах N(B) = 2 происходит событие B: выпадение числа очков кратных трем: выпадение или трех или шести очков. Вероятность такого события .

в) При трех исходах N(C) = 3 происходит событие C: выпадение числа очков больше трех: выпадение четырех, пяти или шести очков. Вероятность этого события .

г) Из шести возможных выпавших чисел четыре (1, 2, 4 и 5) не кратны трем, а остальные два (3 и 6) делятся на три. Значит, интересующее нас событие D,

наступает в четырех случаях, т.е. N(D) = 4. Вероятность такого события: .

Ответ: а) ; б) ; в) ; г) .

Для вычисления вероятности часто используют правило умножения. Для того, чтобы найти число всех возможных исходов независимого проведения двух испытаний А и В, следует перемножить число всех исходов испытания А и число всех исходов испытания В.

Пример. Найдем вероятность того, что при подбрасывании двух костей суммарное число очков окажется равным 5.

Решение. Возможно следующее сочетание очков на первой и второй костях:

1 + 4, 2 + 3, 3 + 2, 4 + 1 – четыре благоприятных случая (N(A) = 4). Всего возможных исходов N = 6·6 = 36 (по шесть для каждой кости). Тогда вероятность рассматриваемого события

Ответ: .

Вероятность Р(А) некоторого события .При решении некоторых задач удобно использовать свойство вероятностей противоположных событий. События А и В называются противоположными, если всякое наступление события А означает ненаступление события В, а ненаступление события А – наступление события В. Событие, противоположное событию А, обозначают символом . Сумма вероятностей противоположных событий равна 1. .

Примеры.

1. Бросаем один раз игральную кость. Событие А – выпадение четного числа очков, тогда событие - выпадение нечетного числа очков.

2. В среднем из 1000 аккумуляторов, поступивших в продажу, 6 неисправны. Найдите вероятность того, что один купленный аккумулятор окажется исправным.

Решение. Элементарный исход – случайно выбранный аккумулятор. Поэтому

N = 1000.

Событию А = {аккумулятор исправен} благоприятствуют 1000 – 6 = 994 исхода.

Поэтому N(A) = 994.

Тогда

Ответ: 0,994.

Эту задачу можно решить с помощью формулы вероятности противоположного события = {аккумулятор неисправен}. Тогда N(Ā)=6.

Имеем = Значит, P(A) = 1- =1 – 0,006 = 0,994.

Ответ: 0,994.

2.3. Решение задач.

1.  Монета бросается два раза. Какова вероятность того, что:
а) герб выпадет хотя бы один раз?      б) герб выпадет два раза? (слайд 13)

Решение. а) Пусть А - событие, состоящее в том, что в результате проведенного испытания герб выпал хотя бы один раз.
Равновозможными элементарными исходами здесь являются: ГГ, ГР, РГ, РР, т.е. N = 4. Событию А благоприятствуют исходы: ГГ, ГР, РГ, т.е. N(A) = 3.
Следовательно, 
б) Пусть В - событие, состоящее в том, что в результате проведенного испытания герб выпал два раза.
Событию В благоприятствует один исход: ГГ, т.е. N(B) = 1.
Следовательно, 

Ответ: а) ; б) .

2.  Игральная кость бросается два раза. Какова вероятность того, что сумма выпавших очков равна 6 (событие А)?

Решение. Равновозможными элементарными исходами здесь являются пары (x, y), где x и y принимают значения: 1,2,3,4,5,6. Таким образом, общее число элементарных исходов равно N = 6 · 6 = 36.
Событию А благоприятствуют пары (1;5), (2;4), (3;3), (4;2), (5;1), число которых равно N(А) = 5.
Следовательно,  .

Ответ: .

3. В ящике лежат 6 красных и 6 синих шаров. Наудачу вынимают 8 шаров. Определите вероятность события А - все выбранные шары красные.

Решение. Р(А) = 0, т.к. это событие А - невозможное.

Ответ: 0.

4. Научная конференция проводится 3 дня. Всего запланировано 50 докладов: в первый день – 30 докладов, а остальные распределены поровну между вторым и третьим днями. Порядок докладов определяется жеребьевкой. Какова вероятность, что доклад профессора М. окажется запланированным на последний день конференции?

Решение. Так как в третий день будут слушать 10 докладов, то благоприятных исходов N(А) = 10, а всего докладов 50, т.е. равновозможных исходов N = 50. Поэтому .

Ответ: 0,2.

5. Перед началом первого тура чемпионата по теннису разбивают на игровые пары случайным образом с помощью жребия. Всего в чемпионате участвует 46 теннисистов, среди которых 19 участников из России, в том числе Ярослав Исаков. Найдите вероятность того, что в первом туре Ярослав Исаков будет играть с каким – либо теннисистом из России. (слайд 16)

Решение. Число всех исходов N = 45. Число элементарных событий, благоприятствующих событию А равно 18. Все элементарные события равновозможны по условию задачи, поэтому

Ответ: 0,4.

3.5. Решение задач в группах. А теперь перейдем к работе в группах. Ваша задача: решить задачи, оформить их в тетрадях и рассказать о проделанной совместной работе. Листочки с заданиями на столах. Помогайте друг другу при решении. (Учитель, в процессе работы учащихся, оказывает помощь каждой группе).

Задачи:

1. Вася, Петя, Коля и Леша бросили жребий - кому начинать игру. Найдите вероятность того, что начинать игру должен будет Петя.

2. Игральный кубик (кость) бросили один раз. Какова вероятность того, что выпало число очков, больше чем 4?

3. В случайном эксперименте бросают два игральных кубика. Найдите вероятность того, что в сумме выпадет 8 очков.

4. В случайном эксперименте монету бросили три раза. Какова вероятность того, что орел выпал ровно два раза?

5. В соревнованиях по толканию ядра участвуют 4 спортсмена из Финляндии, 7 спортсменов из Дании, 9 спортсменов из Швеции и 5- из Норвегии. Порядок, в котором выступают спортсмены, определяется жребием. Найдите вероятность того, что спортсмен, который выступает последним, окажется из Швеции.

6. В чемпионате по гимнастике участвуют 20 спортсменок: 8 из России, 7 из США, остальные – из Китая. Порядок, в котором выступают гимнастки, определяется жребием. Найдите вероятность того, что спортсменка, выступающая первой, окажется из Китая.

7. Фабрика выпускает сумки. В среднем на 100 качественных сумок приходится восемь сумок со скрытыми дефектами. Найдите вероятность того, что купленная сумка окажется качественной. Результат округлите до сотых.

Решения к задачам

1. Случайный эксперимент – бросание жребия. Элементарное событие в этом эксперименте – участник, который выиграл жребий. Перечислим их:

(Вася), (Петя), (Коля) и (Лёша).

Общее число элементарных событий N = 4. Жребий подразумевает, что элементарные события равновозможны. Событию A = {жребий выиграл Петя}

благоприятствует только одно элементарное событие (Петя). Поэтому N(A)=1.

Тогда .

Ответ: 0,25.

2. Случайный эксперимент – бросание кубика. Элементарное событие –число на выпавшей грани. Граней всего шесть. Перечислим все элементарные события: 1,2,3,4,5 и 6. Значит, N=6. Событию A={выпало больше, чем 4} благоприятствует два элементарных события: 5 и 6. Поэтому N(A) = 2. Элементарные события равновозможны, поскольку подразумевается, что кубик честный. Поэтому .

Ответ: .

4. Орёл обозначим буквой О, решку – буквой Р. В описанном эксперименте элементарные исходы – тройки, составленные из букв О и Р. Выпишем все их в таблицу:

Элементарный исход

Число орлов

ООО

3

ООР

2

ОРО

2

ОРР

1

РОО

2

РОР

1

РРО

1

РРР

0

Всего исходов получилось 8. Значит, N=8. Событию А = {орёл выпал ровно два раза} благоприятствует элементарные события ООР, ОРО, РОО. Поэтому N(A)=3. Тогда

Ответ: 0,375.

5. Элементарный исход – спортсмен, который выступает последним. Последним может оказаться любой спортсмен. Всего спортсменов N=4+7+9+5+5=25. Событию А = {последний из Швеции} благоприятствуют только 9 исходов (столько, сколько участвует шведских спортсменов). Поэтому N(A)=9.

Тогда

Ответ: 0,36.

6. Элементарные события – спортсменка, выступающая первой. Поэтому N=20. Чтобы найти число элементарных событий, благоприятствующих событию А = {первой выступает спортсменка из Китая}, нужно подсчитать число спортсменок из Китая: N(A)=20-(8+7)=5. Все элементарные события равновозможны по условию задачи, поэтому

Ответ: 0,25.

7. Элементарный исход – случайно выбранная сумка. Поэтому N = 108.

Событию А = {качественная сумка} благоприятствуют 100 исходов.

Поэтому N(A) = 100.

Тогда

Ответ: 0,93.

8. В ящике 4 красных и 2 синих фломастера. Фломастеры вытаскивают по очереди в случайном порядке. Какова вероятность того, что первый раз синий фломастер появится третьим по счету?

Решение:

Благоприятными будут следующие исходы:

Первый раз – вытащили красный фломастер,

И второй раз – красный,

А третий раз – синий.

Вероятность вытащить красный фломастер (которых в ящике 4) равна 

После этого в ящике остается 5 фломастеров, из них 3 красных, вероятность вытащить красный равна 

Наконец, когда осталось 4 фломастера и из них 2 синих, вероятность вытащить синий равна 

Вероятность события {красный – красный – синий } равна произведению этих вероятностей, то есть 

Ответ: 0,2

9. При подозрении на наличие некоторого заболевания пациента отправляют на ПЦР-тест. Если заболевание действительно есть, то тест подтверждает его в 86 % случаев. Если заболевания нет, то тест выявляет отсутствие заболевания в среднем в 94% случаев.

Известно, что в среднем тест оказывается положительным у 10% пациентов, направленных на тестирование. При обследовании некоторого пациента врач направил его на ПЦР-тест, который оказался положительным. Какова вероятность того, что пациент действительно имеет это заболевание?

Решение:
Уточним условие: «Какова вероятность того, что пациент, ПЦР-тест которого положителен, действительно имеет это заболевание?». В такой формулировке множество возможных исходов - это число пациентов с положительным результатом ПЦР-теста, причем только часть из них действительно заболевшие. Пациент приходит к врачу и делает ПЦР-тест. Он может быть болен этим заболеванием – с вероятностью х. Тогда с вероятностью 1 – х он этим заболеванием не болен.

Анализ пациента может быть положительным по двум причинам:
а) пациент болеет заболеванием, которое нельзя называть, его анализ верен; событие А,
б) пациент не болен этим заболеванием, его анализ ложно-положительный, событие В.
Это несовместные события, и вероятность их суммы равна сумме вероятностей этих событий.

Имеем:

Мы составили уравнение, решив которое, найдем вероятность 

Что такое вероятность х? Это вероятность того, что пациент, пришедший к доктору, действительно болен. Здесь множество возможных исходов - это количество всех пациентов, пришедших к доктору.

Нам же нужно найти вероятность z того, что пациент, ПЦР-тест которого положителен, действительно имеет это заболевание. Вероятность этого события равна   (пациент болен и ПЦР-тест выявил заболевание, произведение событий). С другой стороны, эта вероятность равна   (у пациента положительный результат ПЦР-теста, и при выполнении этого условия он действительно болен).

Получим:   отсюда z = 0,43.
Ответ: 0,43

Вероятность того, что пациент с положительным результатом ПЦР-теста действительно болен, меньше половины!
Кстати, это реальная проблема для диагностики в медицине, то есть в задаче отражена вполне жизненная ситуация.

10. В викторине участвуют 6 команд. Все команды разной силы, и в каждой встрече выигрывает та команда, которая сильнее. В первом раунде встречаются две случайно выбранные команды.

Ничья невозможна. Проигравшая команда выбывает из викторины, а победившая команда играет со следующим случайно выбранным соперником. Известно, что в первых трех играх победила команда А. Какова вероятность того, что эта команда выиграет следующий раунд?

Решение:

Пусть силы команд равны 1, 2, 3, 4, 5 и 6.
В трех раундах участвуют 4 команды, то есть выбирается 4 числа из 6 и среди этих четырех находится наибольшее.
Выпишем в порядке возрастания, какие 4 команды могли участвовать в первых трех раундах:

1234, 1235, 1236, 1245, 1246, 1256, 1345, 1346, 1356, 1456, 2345, 2346, 2356, 2456, 3456 - всего 15 вариантов.

Среди этих 15 групп есть только одна, в которой 4 - наибольшее число. Это группа 1234. Однако, если команда 4 победила команды 1, 2 и 3, то у нее нет шансов выиграть в следующем раунде у команды 5 или 6.

Есть также 4 группы, в которых 5 - наибольшее число. Вероятность того, что команда 5 победила в трех первых раундах, равна   В следующем туре команда 5 встретится либо с командой 6 (и проиграет), либо с командой 1, 2, 3 или 4 и выиграет, то есть в четвертном раунде команда 5 побеждает с вероятностью 

Есть также 10 групп, где 6 - наибольшее число. Вероятность того, что команда 6 победила в трех первых раундах, равна   В четвертом туре команда 6 побеждает с вероятностью 1 (она самая сильная). Соответственно, в следующем туре команда 6 побеждает с вероятностью 1.
Получается   - вероятность команды, победившей в 3 первых турах, победить в четвертном.

Ответ: 


Отчет групп о проделанной работе

4. Итоги урока

Ученики проговаривают, что нового узнали на уроке. Учитель оценивает работу ребят. При выходе из кабинета каждый ученик выбирает прямоугольник по цвету, соответствующему надписями «всё понятно и усвоено», «трудно и не всё понятно», «не понятно и не усвоено», и опускает в соответствующий конверт.

5. Домашнее задание. Решение десяти типовых задач на теорию вероятности.

1. В магазине стоят два платёжных автомата. Каждый из них может быть неисправен с вероятностью 0,05 независимо от другого автомата. Найдите вероятность того, что хотя бы один автомат исправен.

2.  Ковбой Джон попадает в муху на стене с вероятностью 0,9, если стреляет из пристрелянного револьвера. Если Джон стреляет из непристрелянного револьвера, то он попадает в муху с вероятностью 0,2. На столе лежит 10 револьверов, из них только 4 пристрелянные. Ковбой Джон видит на стене муху, наудачу хватает первый попавшийся револьвер и стреляет в муху. Найдите вероятность того, что Джон промахнётся.

3.  Две фабрики выпускают одинаковые стекла для автомобильных фар. Первая фабрика выпускает 45% этих стекол, вторая — 55%. Первая фабрика выпускает 3% бракованных стекол, а вторая — 1%. Найдите вероятность того, что случайно купленное в магазине стекло окажется бракованным.

4.  Всем пациентам с подозрением на гепатит делают анализ крови. Если анализ выявляет гепатит, то результат анализа называется положительным. У больных гепатитом пациентов анализ даёт положительный результат с вероятностью 0,9. Если пациент не болен гепатитом, то анализ может дать ложный положительный результат с вероятностью 0,01. Известно, что 5% пациентов, поступающих с подозрением на гепатит, действительно больны гепатитом. Найдите вероятность того, что результат анализа у пациента, поступившего в клинику с подозрением на гепатит, будет положительным.

5.  Автоматическая линия изготавливает батарейки. Вероятность того, что готовая батарейка неисправна, равна 0,02. Перед упаковкой каждая батарейка проходит систему контроля. Вероятность того, что система забракует неисправную батарейку, равна 0,99. Вероятность того, что система по ошибке забракует исправную батарейку, равна 0,01. Найдите вероятность того, что случайно выбранная изготовленная батарейка будет забракована системой контроля.

6.  Агрофирма закупает куриные яйца в двух домашних хозяйствах. 40% яиц из первого хозяйства — яйца высшей категории, а из второго хозяйства — 20% яиц высшей категории. Всего высшую категорию получает 35% яиц. Найдите вероятность того, что яйцо, купленное у этой агрофирмы, окажется из первого хозяйства.

7.  В торговом центре два одинаковых автомата продают кофе. Вероятность того, что к концу дня в автомате закончится кофе, равна 0,3. Вероятность того, что кофе закончится в обоих автоматах, равна 0,12. Найдите вероятность того, что к концу дня кофе останется в обоих автоматах.

8.  Чтобы поступить в институт на специальность «Лингвистика», абитуриент должен набрать на ЕГЭ не менее 70 баллов по каждому из трёх предметов — математика, русский язык и иностранный язык. Чтобы поступить на специальность «Коммерция», нужно набрать не менее 70 баллов по каждому из трёх предметов — математика, русский язык и обществознание.

Вероятность того, что абитуриент З. получит не менее 70 баллов по математике, равна 0,6, по русскому языку — 0,8, по иностранному языку — 0,7 и по обществознанию — 0,5.

Найдите вероятность того, что З. сможет поступить хотя бы на одну из двух упомянутых специальностей.

9.  Из районного центра в деревню ежедневно ходит автобус. Вероятность того, что в понедельник в автобусе окажется меньше 20 пассажиров, равна 0,94. Вероятность того, что окажется меньше 15 пассажиров, равна 0,56. Найдите вероятность того, что число пассажиров будет от 15 до 19.

10. Вероятность того, что на тестировании по биологии учащийся О. верно решит больше 11 задач, равна 0,67. Вероятность того, что О. верно решит больше 10 задач, равна 0,74. Найдите вероятность того, что О. верно решит ровно 11 задач.

Ответы

вопроса

1

2

3

4

5

6

7

8

9

10

Ответ

0,9975

0,52

0,019

0,0545

0,0296

0,75

0,52

0,408

0,38

0,07


Задача 1.

Решение. Событие, состоящее в том, что исправен хотя бы один автомат является суммой трех несовместных событий, каждое из которых является произведением двух независимых событий:

     А = исправен первый автомат, при этом неисправен второй;

     B = исправен второй автомат, при этом неисправен первый;

     С = исправен первый автомат, при этом второй тоже исправен.

Поэтому для искомой вероятности получаем:

P(A + B+ С) = P(A) + P(B) + P(С) = 0,95 ·0,05  + 0,95 · 0,05 + 0,95 · 0,95 = 0,9975.

Задача 2.

Решение. Джон попадает в муху, если схватит пристрелянный револьвер и попадет из него, или если схватит непристрелянный револьвер и попадает из него. По формуле условной вероятности, вероятности этих событий равны соответственно 0,4·0,9 = 0,36 и 0,6·0,2 = 0,12. События схватить пристрелянный или непристрелянный револьвер образуют полную группу, поэтому по формуле полной вероятности получаем: 0,36 + 0,12 = 0,48. Событие, состоящее в том, что Джон промахнется, противоположное. Его вероятность равна 1 − 0,48 = 0,52.

Задача 3.

Решение. Вероятность того, что стекло сделано на первой фабрике и оно бракованное: 0,45 · 0,03 = 0,0135.

Вероятность того, что стекло сделано на второй фабрике и оно бракованное: 0,55 · 0,01 = 0,0055.

Поэтому по формуле полной вероятности вероятность того, что случайно купленное в магазине стекло окажется бракованным равна 0,0135 + 0,0055 = 0,019.

Задача 4.

Решение. Анализ пациента может быть положительным по двум причинам: А) пациент болеет гепатитом, его анализ верен; B) пациент не болеет гепатитом, его анализ ложен. По формуле условной вероятности, вероятности этих событий равны соответственно  0,9*0,5=0,0454 и 0,01*0,95=0,0095

События быть больным или быть здоровым образуют полную группу (они несовместны и одно из них непременно наступает), поэтому можно применить формулу полной вероятности. Получим: 0,045+0,0095=0,0545.

Задача 5.

Решение. Ситуация, при которой батарейка будет забракована, может сложиться в результате следующих событий: батарейка действительно неисправна и забракована справедливо или батарейка исправна, но по ошибке забракована. По формуле условной вероятности, вероятности этих событий равны соответственно 0,02*0,99  и  0,98*0,01.

События быть неисправной батарейкой или быть исправной образуют полную группу (они несовместны и одно из них непременно происходит), поэтому можно применить формулу полной вероятности. Получим: 0,0198+0,0098=0,0296.

Задача 6.

Решение. Пусть событие A состоит в том, что яйцо имеет высшую категорию, события В1 и В2 состоят в том, что яйцо произведено в первом и втором хозяйствах соответственно. Тогда события А|B1 и А|B2   — события, состоящие в том, что яйцо высшей категории произведено в первом и втором хозяйстве соответственно. По формуле полной вероятности, вероятность того, что будет куплено яйцо высшей категории, равна:

P(AB1)+P(AB2)=P(A|B1)*P(B1)+P(A|B2)*P(B2)=0,4*P(B1)+0,2(1-P(B1))=0,2P(B1)+0,2

По условию эта вероятность равна 0,35, поэтому для вероятности того, что купленное яйцо произведено в первом хозяйстве имеем: P(B1)=(0,35-0,2):0,2=0,75

Задача 7.

Решение. Рассмотрим события

А = кофе закончится в первом автомате,

В = кофе закончится во втором автомате.

Тогда

A·B = кофе закончится в обоих автоматах,

A + B = кофе закончится хотя бы в одном автомате.

По условию P(A) = P(B) = 0,3; P(A·B) = 0,12.

События A и B совместные, вероятность суммы двух совместных событий равна сумме вероятностей этих событий, уменьшенной на вероятность их произведения:

P(A + B) = P(A) + P(B) − P(A·B) = 0,3 + 0,3 − 0,12 = 0,48.

Следовательно, вероятность противоположного события, состоящего в том, что кофе останется в обоих автоматах, равна 1 − 0,48 = 0,52.

Задача 8.

Решение. Есть три варианта поступления абитуриента хотя бы на одну специальность:

а) поступить на лингвистику при этом не поступив на коммерцию: вероятность 0,6 · 0,8 · 0,7 · 0,5;

б) поступить и на лингвистику, и на коммерцию: вероятность 0,6 · 0,8 · 0,7 · 0,5;

в) не поступить на лингвистику, при этом поступив на коммерцию: вероятность 0,6 ·  0,8 · 0,3 · 0,5.

Эти события несовместные, искомая вероятность суммы этих событий равна сумме их вероятностей:

0,6 · 0,8 · (0,35 + 0,35 + 0,15) = 0,48 · 0,85 = 0,408.

Задача 9.

Решение. Рассмотрим события A = «в автобусе меньше 15 пассажиров» и В = «в автобусе от 15 до 19 пассажиров». Их сумма — событие A + B = «в автобусе меньше 20 пассажиров». События A и В несовместные, вероятность их суммы равна сумме вероятностей этих событий:

P(A + B) = P(A) + P(B). 

Тогда, используя данные задачи, получаем: 0,94 = 0,56 + P(В), откуда P(В) = 0,94 − 0,56 = 0,38.

 

Задача 10.

Решение. Рассмотрим события A = «учащийся решит 11 задач» и В = «учащийся решит больше 11 задач». Их сумма — событие A + B = «учащийся решит больше 10 задач». События A и В несовместные, вероятность их суммы равна сумме вероятностей этих событий:

P(A + B) = P(A) + P(B).

Тогда, используя данные задачи, получаем: 0,74 = P(A) + 0,67, откуда P(A) = 0,74 − 0,67 = 0,07.


Тема урока «Решение ключевых задач по теме «Геометрическая вероятность».

Тип урока: урок обобщения и систематизации знаний.

Оборудование: доска, проектор, компьютер, карточки, лист самооценки ученика.

Цель урока: формировать способность к обобщению знаний, учащихся о методах решения вероятностных задач; способствовать дальнейшей отработке знаний и умений применять общие методы при решении задач.

Задачи урока:

образовательные:

- содействовать дальнейшей отработке умения решать вероятностные задачи различной сложности, используя тот или иной общий метод решения;

- дальнейшее формирование умения обобщать и делать выводы.

развивающие:

- развитие логического мышления, умения анализировать учебный материал;

- умение работать в группе и индивидуально, развитие умения преодолевать трудности при решении математических задач.

воспитательные:

- выработка привычки к постоянной занятости, воспитание отзывчивости, взаимопомощи, трудолюбия, аккуратности, силы воли.

Результаты:

- знать правила вычисления вероятности случайных событий, геометрической вероятности; применять их при решении практических задач;

- знать универсальный характер законов логики математических рассуждений, их применимость в различных областях человеческой деятельности;

- определять вероятностный характер различных процессов и закономерностей окружающего мира;

- уметь самостоятельно ставить новые учебные задачи путем задавания вопросов о неизвестном;

- уметь структурировать информации в виде записи выводов и определений;

- уметь правильно излагать свои мысли, понимать смысл поставленной задачи.

Ход урока:

1. Сообщение целей урока и его плана.

2. Актуализация опорных знаний и умения, проверка дом. задания.

а) ответы на вопросы по домашней работе по теме (6-7 минут);

б) устная работа по вопросам теории, заданным также на дом:

Вопросы:

  1. Какие виды событий мы знаем?

  2. Чему равна вероятность достоверного события?

  3. С какими событиями вы работали, решая дом. задачи?

Ответы:

  1. Различают следующие виды случайных событий: достоверные, невозможные и случайные. События обозначаются большими латинскими буквами А, В, С,...,Z. Достоверное событие всегда происходит в результате наблюдения или испытания. Достоверное событие обозначается символом – W.

  2. Статистическая вероятность достоверного события равна единице, т. е. P(N) = 1.

  3. Ученики отвечают самостоятельно.

3. Систематизация знаний и умений с использованием заранее заготовленных заданий.

Сегодня нам предстоит отрабатывать определение геометрической вероятности в ходе решения ключевых задач. Согласитесь, что не всегда мы будем решать задачи, в которых будет известно, кто и сколько шаров и какого цвета и из какого ящика достаёт. Так же не всегда надо будет найти вероятность попадания в мишень, нам, например, надо будет найти вероятность попадания в определенную часть мишени. Классическое определение вероятности позволяет найти её по формуле: Р(А)=  , где n – количество благоприятных исходов, n – количество общих исходов.

Это определение предполагает, что количество исходов испытаний конечно, т.е. может быть большое, но определенное число. Но не всегда мы можем при решении задач его получить.

Например, на отрезке ВС наудачу поставили точку А.

Вопрос №1: «Каково количество общих исходов? Сколько точек можно в принципе отметить на рисунке?» (даётся время учащимся, чтобы высказать свою точку зрения).

Ответ на него очень простой, но использовать его при решении задачи нельзя, потому что бесконечность – это сколько? Решать такие и аналогичные задачи нам поможет понятие геометрической вероятности. Само название говорит о многом. Геометрическая вероятность – что-то, связанное с геометрией. Не просто связанное, а очень даже зависящее от неё, а также с некоторыми понятиями алгебры, такими как построение графиков. Рассмотрим задачи из предложенных на рабочих листах (см. приложение).

(Ученики работают по группам, обсуждают ход решения второй и третьей из предложенных задач).

Задача 2. В прямоугольник 5*4 см2 вписан круг радиуса 1,5 см. Какова вероятность того, что точка, случайным образом поставленная в прямоугольник, окажется внутри круга?

Решение: По определению геометрической вероятности искомая вероятность равна отношению площади круга (в который точка должна попасть) к площади прямоугольника (в которой точка ставится), т.е.

P=Sкруга/Sпрямоугольника=π⋅1,52/5⋅4=0,353.

Ответ: 0,353

Задача 3. В круг радиуса R наудачу брошена точка. Найдите вероятность того, что эта точка окажется внутри данного вписанного правильного треугольника.

Решение: Искомая вероятность равна отношению площади треугольника к площади круга:

P= 

Ответ: 

Примечание. При оформлении задач следует обязательно указывать размерность (единицы, метры, квадратные единицы, квадратные метры и т.д.). На финальном этапе вычислений геометрическая мера сокращается. В результате чего получилась привычная безразмерная вероятность.

(подробный разбор хода решения задачи №4 и оформление на доске)

Задача 4. Из отрезка [0, 2] на удачу выбраны два числа х и у. Найдите вероятность того, что эти числа удовлетворяют неравенствам х2≤4у≤4х.

Решение: По условиям опыта координаты точки (х,у) удовлетворяют системе неравенств:

0 ≤х≤2

0≤у≤2.

Это значит, что точка (х,у) наудачу выбирается из множества точек квадрата со стороной 2. Интересующее нас событие происходит в том и только в том случае, когда выбранная точка (х,у) окажется под прямой и над параболой. Эта область получена как множество точек, ординаты которых удовлетворяют неравенствам х2≤4у≤4х. Следовательно, искомая вероятность равна отношению площади области к площади квадрата:

Р= 

Ответ:  

Задача 5. Буратино посадил в центре прямоугольного листа бумаги размером 20 см на 25 см круглую кляксу радиусом 1 см. Сразу после этого Буратино посадил еще одну такую же кляксу, которая также целиком оказалась на листе. Найдите вероятность того, что эти две кляксы не соприкасаются.

Решение:

Первая клякса, радиусом 1 см, закрашена красным цветом.

Контурами показаны возможные расположения второй кляксы - в случае касания первой и второй. Видим, что кляксы касаются тогда, когда вторая попадет в кольцо, образованное окружностью радиусом 3 см и окружностью радиусом 1 см. Кляксы не должны также пересекаться. Значит, вторая клякса не должна попасть в круг, радиусом 3. Найдем площадь круга.

S круга = п*32 = 9п см2.

Благоприятным считаем исход, когда кляксы не имеют общих точек.

В этом случае область для попадания - прямоугольник с вырезанным кругом. Найдем площадь этой фигуры S1.

S1 = 20*25 - 9п = 500-9п. Вероятность Р = S1 / S прямоугольника = (500-9*3,14) / 500 ≈ 0,94.

Ответ: 0,94

Задача 6. В треугольник со сторонами, а=9, в=13, с=16 вписан круг. Точка М произвольно ставится в треугольник. Найти вероятность того, что точка попадёт в круг.

Решение: поскольку точка ставится в треугольник, а круг лежит внутри, то общему числу исходов соответствует площадь треугольника, а множеству благоприятствующих исходов – площадь вписанного круга. Что тут сказать? Ищем площади. Если даны длины сторон треугольника, то его площадь удобно найти по формуле Герона. Площадь вписанного круга найдём по формуле S= . По геометрическому определению: Р(А)=Sкр/SΔ~0,51

Ответ: 0,51

Задача 7. В круге радиуса 10 см находится прямоугольный треугольник с катетами 12 и 7 см. В круг наудачу ставится точка. Найти вероятность того, что она не попадёт в данный треугольник. Следует отметить, что в этой задаче треугольник вовсе не обязан как-то касаться окружности, он просто расположен внутри круга и всё. Будьте внимательны!

Решение.

S₁=πR²=100π

S₂=0,5ab=0,5·12·7=42

P=S₂/S₁=42/(100π)≈0,1337

Ответ: 0,1337

Задача 8. Загадываются два числа х и у в промежутке от 0 до 5. Какова вероятность, что ху2?

Решение: т.к. загадываются 2 произвольных числа от нуля до пяти (они могут быть и иррациональными), то общему количеству исходов соответствует площадь квадрата S=5*5=25 ед.2 Изобразим ветвь гиперболы ху=2 т.е. у = , которая делит квадрат на две части:

Теперь выясним, какой из этих двух «кусков» удовлетворяет неравенству xy2 . Для этого выберем любую точку, не принадлежащую гиперболе, проще всего взять (0;0), и подставим её координаты в наше неравенство. Получено неверное неравенство, а значит, условию xy2 соответствует «верхний кусок», площадь которого вычислим с помощью определённого интеграла. Уточним нижний предел интегрирования аналитически (найдём точку пересечения гиперболы ху = 2 и прямой у=5): 5х=2; х=0,4

S= 

По геометрическому определению: Р(А)=   = 0,72

Ответ: 0,72

4. Итог урока. Рефлексия: заполнить таблицу, записать ответ в третьем столбике

№ п/п

Вопрос

Ответ

1

С каким определением вероятности мы сегодня работали?


2

Что используют при нахождении геометрической вероятности?


3

Что является количеством благоприятных или общих исходов?


4

Кто доволен своей работой?


5

Что на ваш взгляд мешало в работе?


6

Какую оценку вы поставите себе за урок?


Фамилия имя, класс



5. Домашнее задание: решить задачи

Задача 1. Какова вероятность Вашей встречи с другом, если вы договорились встретиться в определенном месте, с 12.00 до 13.00 часов и ждете друг друга в течение 5 минут?

Задача 2. На отрезок АВ длины L, брошена точка М так, что любое ее положение на отрезке равновозможно. Найти вероятность того, что меньший из отрезков (АМ или МВ) имеет длину, большую чем L/3.

Задача 3. Какова вероятность того, что сумма двух наугад взятых положительных чисел, каждое из которых не больше трех, не превзойдет трех, а их произведение будет не больше 2/7?

Задача 4. Наудачу взяты два положительных числа х и у, каждое из которых не превышает единицы. Найти вероятность того, что сумма х + у не превышает единицы, а произведение ху не меньше 0,09.

Задача 5. На отрезке АВ длиной l независимо одна от другой поставлены 2 точки L и M, положение каждой из которых равновозможно на AB. Найти вероятность того, что точка L будет ближе к точке M, чем к точке A.

Задача 6. Моменты начала двух событий наудачу распределены в промежутке времени от T1 до T2. Одно из событий длится 10 мин., другое – t мин. Определить вероятность того, что: а) события «перекрываются» по времени; б) «не перекрываются».
T1=1100; T2=1300; t=15.

Задача 1. Ответ: 0,16

Задача 2. Решение: Разбиваем отрезок AB длины L числовой оси точками X1, X2 на 3 одинаковые части (отрезков), каждый из которых имеет длину L/3. Если точка MM не попадет в отрезок AX1 или X2B, то выполнится условие задачи (меньший из отрезков AM и MB имеет длину, большую L/3). Следовательно, искомая вероятность равна отношению длины центрального отрезка X1X2 к длине всего отрезка L: P=(L/3)/L=1/3.

Задача 3. Ответ: 0,14

Задача 4. Ответ: 0,202

Задача 5. Ответ: 0,75

Задача 6. Ответ: 0,197; 0,803.

Заключение

В процессе выполнения выпускной квалификационной работы были рассмотрены такие разделы: история развития теории вероятности, основные понятия теории вероятности, методика обучения теории вероятности в 10-11 классах средней школы, разработан элективный курс и несколько тематических уроков.

Экспериментальное преподавание проводилось в школе № 438 п. Лисий Нос Приморского района Ленинградской области среди учащихся 10 классов, посещавших факультативный курс. Достаточно высокий уровень усвоения знаний учащихся позволяет судить об эффективности занятий элективного курса при обучении теории вероятностей в старших классах общеобразовательной школы.

Таким образом в результате выполнения выпускной квалификационной работы поставленная цель достигнута, задачи выполнены.

Перспектива дальнейшего применения материала выпускной квалификационной работы состоит в том, что она может быть использована в качестве дополнительного пособия при ознакомлении с методикой преподавания основ теорией вероятностей в средней школе как ученика и учителям математики общеобразовательных школ при обучении теории вероятностей.

В качестве основной цели опытно - экспериментальной работы была представлена апробация предложенных методических рекомендаций по изучению основных теоретико-вероятностных вопросов в школьном курсе математики в классах с базовым изучением математике.

Достижение поставленной цели потребовало решения следующих задач:

- разработать содержание цикла уроков по теории вероятностей;

- проверить целесообразность разработанных методических рекомендаций.

Основная гипотеза опытной работы: включение элементов в теории вероятностей в математическую подготовку учащихся способствует общему повышению интеллектуального уровня учащихся и качества их математической подготовки.

При проведении опытной работы пользовались следующими методами:

- наблюдение за процессом усвоения знаний учащимися;

- беседы с учителем математики этого класса и учениками;

- проведение диагностической контрольной работы;

- количественная и качественная обработка полученных данных.

Эксперимент был проведен в школе № 438 п. Лисий Нос Приморского района Ленинградской области среди учащихся 10 класса. В классе 11 учащихся. Большинство учащихся (6 из 11 учеников) справилось с работой на "отлично"59%, 45% - "на хорошо" (5 из 11 учеников), остальные 10% - "на удовлетворительно" (что составляет 1 ученик из 11). Задания для апробации были использованы на основе разработанных заданий из базы данных по экзаменационным задачам на теорию вероятности, задания приведены в приложении.

Анализируя результаты работы учеников, можно сделать вывод, что большая часть учащихся усвоила основные теоретико-вероятностные вопросы и умеет решать задачи с применением классического определения вероятности. Такие результаты возможно связанны с применением в процессе обучения разработанных методических рекомендаций.





Литература

  1. Виленкин Н. Я. Популярная комбинаторика. - М.: Наука, 1975. Коваленко И.Н., Филиппова А.А. Теория вероятностей и математическая статистика. - М., 1973.

  2. Пугачев B.C. Теория вероятностей и математическая статистика. - М. Физматлит, 1979.

  3. Четыркин Е.М., Калахман И.Л. Вероятность и статистика. - М., 1982.

  4. Мордкович А.Г., Семенов П.В. События. Вероятность. Статистика:

Дополнительные материалы к курсу алгебры для 7 - 9 кл. - М. Мнемозина, 2002. (к учебникам А.Г. Мордковича)

  1. Ткачева М.В.,Федорова Н.Е. Алгебра, 7-9: Элементы статистики и вероятность. - М. Просвещение, 2003. (к учебникам А.Ш. Алимова и др.)

  2. Бунимович Е.А., Булычев В.А. Вероятность и статистика, 5 - 9 кл. - М. Дрофа, 2002.

  3. Мордкович А.Г., Семенов П.В. События, вероятности, статистическая обработка данных, - Математика (приложение к газете «Первое сентября»), №34, 35,41, 43, 44, 48, 2002, №11, 17, 2003.

  4. Дынкин Е. Б., Молчанов С. А., Розенталь А. Л. Математические соревнования. Арифметика и алгебра. - М.: Наука, 1998

  5. Слойер К. Математические фантазии. - М.: Мир, 1993.

  6. Тюрин Ю. Н. и др. Теория вероятностей и статистика. - М.: МЦНМО: Московские учебники, 2004.

  7. Горелова Г. В., Кацко И. А. Теория вероятностей и математическая статистика в примерах и задачах с применением Excel. - Ростов н/Д: Феникс, 2006.

  8. Решение задач по статистике, комбинаторике и теории вероятностей.

7-9 классы./ Авт.-сост. В.Н.Студенецкая. Изд.2-е, испр,- Волгоград: Учитель, 2006.

  1. Макарычев Ю.Н., Миндюк Н.Г. Алгебра. Элементы статистики и теории вероятностей. — М.: Просвещение, 2006.

  2. Палий И.А. Введение в теорию вероятностей. - М.: Высшая школа, 2005.

  3. Письменный Д.Т. Конспект лекций по теории вероятностей, математической статистике и случайным процессам. - М.: Айрис пресс, 2006.

  4. Болдырева М.Х., Карпухин Ю.П., Клековкин Г.А. Комбинаторика. Бином Ньютона. Избранные вопросы школьного курса математики, выпуск 7. - Самара: СИПКРО, 2002.

Интернет – ресурсы:

tp://alexlarin.net - различные материалы для подготовки http://www.egetrener.ru - видеоуроки http://www.mathege.ru - открытый банк заданий

http://live.mephist.ru/?mid=1255348015#comments - Открытый банк http://reshuege.ru/

http://egeent.narod.ru/matematika/online/

http://vkontakte.ru/app1841458 - приложение ВКонтакте - отработка части В http://matematika-ege.ru

http://uztest.ru/

http://www.diary.ru/~eek - Математическое сообщество

https://ru.wikipedia.org/wiki/История_теории_вероятностей

https://gigabaza.ru/doc/160214.html

https://ege.sdamgia.ru/test?theme=185

https://www.matburo.ru/ex_tv.php?p1=tvgeom


Разработки некоторых уроков для учащихся 10-11 классов по теории вероятности.

Приложение

Определение вероятности. Простые задачи из вариантов ЕГЭ.

Данные задания были использованы для апробации в 10 классе ГБОУ Школы №438. Разберем задачи по теории вероятностей, входящие в сборники для подготовки к ЕГЭ.

1. В фирме такси в данный момент свободно   машин:   красных,   желтых и 4 зеленых. По вызову выехала одна из машин, случайно оказавшихся ближе всего к заказчице. Найдите вероятность того, что к ней приедет желтое такси.

Всего имеется   машин, то есть к заказчице приедет одна из пятнадцати. Желтых — девять, и значит, вероятность приезда именно желтой машины равна 9/15, то есть 0,6.

2. В сборнике билетов по биологии всего   билетов, в двух из них встречается вопрос о грибах. На экзамене школьнику достаётся один случайно выбранный билет. Найдите вероятность того, что в этом билете не будет вопроса о грибах.

Очевидно, вероятность вытащить билет без вопроса о грибах равна 23/25, то есть 0,92.

3. Родительский комитет закупил 30 пазлов для подарков детям на окончание учебного года, из них 12 с картинами известных художников и 18 с изображениями животных. Подарки распределяются случайным образом. Найдите вероятность того, что Вовочке достанется пазл с животным.

Задача решается аналогично.

Ответ: 0,6.

4. В чемпионате по гимнастике участвуют   спортсменок:   из России,   из США, остальные — из Китая. Порядок, в котором выступают гимнастки, определяется жребием. Найдите вероятность того, что спортсменка, выступающая последней, окажется из Китая.

Давайте представим, что все спортсменки одновременно подошли к шляпе и вытянули из нее бумажки с номерами. Кому-то из них достанется двадцатый номер. Вероятность того, что его вытянет китайская спортсменка, равен 5/20 (поскольку из Китая —   спортсменок). Ответ: 0,25.

5. Ученика попросили назвать число от 1 до 100. Какова вероятность того, что он назовет число кратное пяти?

1, 2, 3, 4, 5, 6, 7, 8, 9, 10, 11 … 100

Каждое пятое число из данного множества делится на 5. Значит, вероятность равна 1/5.

6. Брошена игральная кость. Найдите вероятность того, что выпадет нечетное число очков.

1, 3, 5 — нечетные числа;  2, 4, 6 — четные. Вероятность нечетного числа очков равна 1/2.

Ответ: 0,5.

7. Монета брошена три раза. Какова вероятность двух «орлов» и одной «решки»?

Заметим, что задачу можно сформулировать по-другому: бросили три монеты одновременно. На решение это не повлияет.

Как вы думаете, сколько здесь возможных исходов?

Бросаем монету. У этого действия два возможных исхода: орел и решка

Две монеты — уже четыре исхода:

орел

орел

орел

решка

решка

орел

решка

решка

Три монеты? Правильно,   исходов, так как  .

Вот они:

орел

орел

орел

орел

орел

решка

орел

решка

орел

решка

орел

орел

орел

решка

решка

решка

орел

решка

решка

решка

орел

решка

решка

решка

Два орла и одна решка выпадают в трех случаях из восьми.

Ответ: 3/8.

8. В случайном эксперименте бросают две игральные кости. Найдите вероятность того, что в сумме выпадет   очков. Результат округлите до сотых.

Бросаем первую кость — шесть исходов. И для каждого из них возможны еще шесть — когда мы бросаем вторую кость.

Получаем, что у данного действия — бросания двух игральных костей — всего   возможных исходов, так как  .

А теперь — благоприятные исходы:

 

 

 

 

 

Вероятность выпадения восьми очков равна  .

 Стрелок попадает в цель с вероятностью  . Найдите вероятность того, что он попадёт в цель четыре раза выстрела подряд.

Если вероятность попадания равна   — следовательно, вероятность промаха  . Рассуждаем так же, как и в предыдущей задаче. Вероятность двух попадания подряд равна  . А вероятность четырех попаданий подряд равна  .

Вероятность: логика перебора.

 В кармане у Пети было   монеты по   рублей и   монеты по   рублей. Петя, не глядя, переложил какие-то   монеты в другой карман. Найдите вероятность того, что пятирублевые монеты лежат теперь в разных карманах.

Мы знаем, что вероятность события равна отношению числа благоприятных исходов к общему числу исходов. Но как посчитать все эти исходы?

Можно, конечно, обозначить пятирублевые монеты цифрами  , а десятирублевые цифрами   — а затем посчитать, сколькими способами можно выбрать три элемента из набора  .

Однако есть более простое решение:

Кодируем монеты числами:   (это пятирублёвые),   (это десятирублёвые). Условие задачи можно теперь сформулировать так:

Есть шесть фишек с номерами от   до  . Сколькими способами можно разложить их по двум карманам поровну, так чтобы фишки с номерами   и   не оказались вместе?

Давайте запишем, что у нас в первом кармане.

Для этого составим все возможные комбинации из набора  . Набор из трёх фишек будет трёхзначным числом. Очевидно, что в наших условиях   и   — это один и тот же набор фишек. Чтобы ничего не пропустить и не повториться, располагаем соответствующие трехзначные числа по возрастанию:

123, 124, 125, 126 …

А дальше? Мы же говорили, что располагаем числа по возрастанию. Значит, следующее —  , а затем:

.

Все! Мы перебрали все возможные комбинации, начинающиеся на  . Продолжаем:

.

Всего   возможных исходов.

У нас есть условие — фишки с номерами   и   не должны оказаться вместе. Это значит, например, что комбинация   нам не подходит — она означает, что фишки   и   обе оказались в не в первом, а во втором кармане. Благоприятные для нас исходы — такие, где есть либо только  , либо только  . Вот они:

134, 135, 136, 145, 146, 156, 234, 235, 236, 245, 246, 256 – всего благоприятных исходов.

Тогда искомая вероятность равна  .

Ответ:  .

Сумма событий, произведение событий и их комбинации

 Вероятность того, что новый электрический чайник прослужит больше года, равна 0,93. Вероятность того, что он прослужит больше двух лет, равна 0,87. Найдите вероятность того, что он прослужит меньше двух лет, но больше года.

Проработав год, чайник может либо сломаться на второй год, либо благополучно служить и после 2 лет работы.
Пусть   – вероятность того, что чайник прослужил больше года.

 – вероятность того, что он сломается на второй год,   – вероятность того, что он прослужит больше двух лет. Очевидно,  Тогда 

Ответ: 0,06

События, взаимоисключающие друг друга в рамках данной задачи, называются несовместными. Появление одного из несовместных событий исключает появление других.

Сумма двух событий – термин, означающий, что произошло или первое событие, или второе, или оба сразу.

Вероятность суммы несовместных событий равна сумме их вероятностей.

В нашей задаче события «чайник сломался на второй год работы» и «чайник работает больше двух лет» - несовместные. Чайник или сломался, или остается в рабочем состоянии.

12. (А) Два грузовика, работая совместно, вывозят снег с улицы Нижняя Подгорная, причем первый грузовик должен сделать три рейса с грузом снега, а второй - два. Вероятность застрять с грузом снега при подъеме в горку равна 0,2 для первого грузовика и 0,25 - для второго. С какой вероятностью грузовики вывезут снег с улицы Нижняя Подгорная, ни разу не застряв на горке?

Вероятность для первого грузовика благополучно одолеть горку   Для второго   Поскольку первый грузовик должен сделать 3 рейса

, а второй – два, грузовики ни разу не застрянут на горке с вероятностью 

 Агрофирма закупает куриные яйца в двух домашних хозяйствах. 40% яиц из первого хозяйства — яйца высшей категории, а из второго хозяйства — 20% яиц высшей категории. Всего высшую категорию получает 35% яиц. Найдите вероятность того, что яйцо, купленное у этой агрофирмы, окажется из первого хозяйства.

Нарисуем все возможные исходы ситуации. Покупатель пришел в магазин, который принадлежит агрофирме, и купил яйцо. Надо найти вероятность того, что это яйцо из первого хозяйства.

Яйца могут быть только или из первого домашнего хозяйства, или из второго, причем эти два события несовместны. Других яиц в этот магазин не поступает.

Пусть вероятность того, что купленное яйцо из первого хозяйства, равна  . Тогда вероятность того, что яйцо из второго хозяйства (противоположного события), равна  .

Яйца могут быть высшей категории и не высшей. В первом хозяйстве 40% яиц имеют высшую категорию, а 60% - не высшую. Это значит, что случайно выбранное яйцо из первого хозяйства с вероятностью 40% будет высшей категории.

Во втором хозяйстве 20% яиц высшей категории, а 80% - не высшей.

Пусть случайно выбранное в магазине яйцо - из первого хозяйства и высшей категории. Вероятность этого события равна произведению вероятностей: 

Вероятность того, что яйцо из второго хозяйства и высшей категории, равна 

Если мы сложим эти две вероятности, мы получим вероятность того, что яйцо имеет высшую категорию. По условию, высшую категорию имеют 35% яиц, значит, эта вероятность равна 0,35.

Мы получили уравнение:

Решаем это уравнение и находим, что   – вероятность того, что яйцо, купленное у этой агрофирмы, оказалось из первого хозяйства.

 Всем пациентам с подозрением на гепатит делают анализ крови. Если анализ выявляет гепатит, то результат анализа называется положительным. У больных гепатитом пациентов анализ даёт положительный результат с вероятностью 0,9. Если пациент не болен гепатитом, то анализ может дать ложный положительный результат с вероятностью 0,01. Известно, что 5% пациентов, поступающих с подозрением на гепатит, действительно больны гепатитом. Найдите вероятность того, что результат анализа у пациента, поступившего в клинику с подозрением на гепатит, будет положительным.

С чем пришел пациент в клинику? – С подозрением на гепатит. Возможно, он действительно болен гепатитом, а возможно, у его плохого самочувствия другая причина. Может быть, он просто съел что-нибудь.

Вероятность того, что он болен гепатитом, равна 0,05 (то есть 5%). Вероятность того, что он здоров, равна 0,95 (то есть 95%).

Пациенту делают анализ. Покажем на схеме все возможные исходы:

Если он болен гепатитом, анализ дает положительный результат с вероятностью 0,9. То есть анализ покажет: «есть гепатит».
Заметим, что анализ не во всех случаях выявляет гепатит у того, кто действительно им болен. С вероятностью 0,1 анализ не распознает гепатит у больного.

Более того. Анализ может ошибочно дать положительный результат у того, кто не болеет гепатитом. Вероятность такого ложного положительного результата 0,01. Тогда с вероятностью 0,99 анализ даст отрицательный результат, если человек здоров.

Найдем вероятность того, что результат анализа у пациента, поступившего в клинику с подозрением на гепатит, будет положительным.

Благоприятные для этой ситуации исходы: человек болен, и анализ положительный (вероятность одновременного наступления этих двух событий равна  ), или человек здоров, и анализ ложный положительный (вероятность одновременного наступления этих двух событий равна  ). Так как события «человек болен» и «человек не болен» несовместны, то вероятность того, что результат анализа будет положительным, равна 

Ответ: 0,0545.

 Чтобы поступить в институт на специальность «Лингвистика», абитуриент З. должен набрать на ЕГЭ не менее 70 баллов по каждому из трёх предметов — математика, русский язык и иностранный язык. Чтобы поступить на на специальность «Коммерция», нужно набрать не менее 70 баллов по каждому из трёх предметов — математика, русский язык и обществознание.

Вероятность того, что абитуриент З. получит не менее 70 баллов по математике, равна 0,6, по русскому языку — 0,8, по иностранному языку — 0,7 и по обществознанию — 0,5.
Найдите вероятность того, что З. сможет поступить хотя бы на одну из двух упомянутых специальностей.

Заметим, что в задаче не спрашивается, будет ли абитуриент по фамилии З. учиться и лингвистике, и коммерции сразу и получать два диплома. Здесь надо найти вероятность того, что З. сможет поступить хотя бы на одну из двух данных специальностей – то есть наберет необходимое количество баллов.

Для того чтобы поступить хотя бы на одну из двух специальностей, З. должен набрать не менее 70 баллов по математике. И по русскому. И еще – обществознания или иностранный.
Вероятность набрать 70 баллов по математике для него равна 0,6.
Вероятность набрать баллы по математике и русскому равна 

Разберемся с иностранным и обществознанием. Нам подходят варианты, когда абитуриент набрал баллы по обществознанию, по иностранному или по обоим. Не подходит вариант, когда ни по языку, ни по «обществу» он не набрал баллов. Значит, вероятность сдать обществознание или иностранный не ниже чем на 70 баллов равна

В результате вероятность сдать математику, русский и обществознание или иностранный равна   Это ответ.






 











































2




Скачать

Рекомендуем курсы ПК и ППК для учителей

Вебинар для учителей

Свидетельство об участии БЕСПЛАТНО!